Data Sufficiency Flashcards

1
Q

What are the two strategies for eliminating answer choices for data sufficiency problems?

A

If you are starting with statement (1), then write down the following grid and work from top to bottom, left to right.
AD
BCE

If you are starting with statement (2), then write down the following grid and work from top to bottom, left to right.
BD
ACE

How well did you know this?
1
Not at all
2
3
4
5
Perfectly
2
Q

What are the possible answer choices for data sufficiency problems?

A

12TEN

A. (1) ALONE is sufficient, but statement (2) alone is not sufficient.
B. (2) ALONE is sufficient, but statement (1) alone is not sufficient.
C. Together or Combined sufficient
D. Either statement ALONE is sufficient
E. Neither statement is sufficient

How well did you know this?
1
Not at all
2
3
4
5
Perfectly
3
Q

What are the steps to solving a data sufficiency problem?

A

(1) Separate additional info from the actual question
(2) Determine whether the question is Value or Yes/No
- Value: the question asks for the value of an unknown (e.g. what is x?)
a. Statement is sufficient when it provides one possible value
b. Statement is not sufficient when it provides more than one possible value
- Yes/No: the question that is asked has two possible answers, yes or no (e.g. is x even?)
a. Statement is sufficient when it provides a definite Yes or definite No
b. Statement is not sufficient when the answer could be Yes or No
(3) Decide exactly what the question is asking
(4) Use the grid to evaluate the statements

How well did you know this?
1
Not at all
2
3
4
5
Perfectly
4
Q

What is the three step method for Yes/No data sufficiency questions?

A

(1) C. Criteria. Generate your criteria from statement 1, statement 2, and both clues together; be sure to always include the criteria from the question stem
(2) L. List. Possible values that meet the criteria. Go through the integers in order, starting at 0 if possible. Try to list 5 values.* If you cannot find 5 values that meet the criteria, try to list at least 2
(3) A. Answer. Mark each value in your list with a “Yes” or a “No”. If all of the values in your list yield the same answer, then the statement is sufficient. If, on the other hand, the values in your list yield different answers – some a “Yes” and some a “No” – then the clue is not sufficient

  • Two exceptions to the rule: NPZ and FIZ
  • Note: It is recommended to list five values; however, if it gets to the point where it is difficult to list five values, then try to list at least two and see what happens
How well did you know this?
1
Not at all
2
3
4
5
Perfectly
5
Q

What are the two exceptions to the three step method for Yes/No data sufficiency questions?

A

(1) NPZ: Questions that ask positive vs. negative. Try to list three values: negative, positive, zero
(2) FIZ: Questions that ask integer vs. fraction. Try to list three values: fraction, integer, zero

*NOTE: If you are only able to list one value for NPZ or FIZ, then we have a definitive “Yes” or “No”

How well did you know this?
1
Not at all
2
3
4
5
Perfectly
6
Q

When should you use the three step method for Yes/No data sufficiency questions?

A

Useful for yes/no questions that focus or hinge on ONE variable

Examples:

  • Is integer x prime? (Regular)
  • Is x < 0? (NPZ)
  • Is the value of x between 0 and 1? (FIZ)
  • Is x an integer? (FIZ)
How well did you know this?
1
Not at all
2
3
4
5
Perfectly
7
Q

What are the benefits of the three step method for Yes/No data sufficiency questions?

A

(1) Rock-solid approach for yes/no questions that hinge on a single variable
(2) Uses real numbers and thus eliminates abstraction

*CAVEAT: the testing method is NOT always the most efficient method; there might be more mathematically efficient methods; but the testing method is an extremely useful “FALL-BACK” strategy when you do not “see” a more efficient method?

How well did you know this?
1
Not at all
2
3
4
5
Perfectly
8
Q

If S is the infinite sequence S(1) = 9, S(2) = 99, S(3) = 999…S(k) = 10^k – 1, is every term in S divisible by the prime number p?

(1) p is greater than 2
(2) At least one term in sequence S is divisible by p

A

OG10, Lab # 2, Q 135. Answer E. The question does not require rephrasing. Apply the three step method for Yes/No testing (Regular question).
(1) INSUFFICIENT. We get a yes and no to two of our chosen values
Criteria: p = prime, p > 2
Values: 3, 5, 7, 11, 13
Answer: Y N
(2) INSUFFICIENT. We get a yes and no to two of our chosen values
Criteria: p = prime, at least one term is divisible by p
Values: 3, 11
Answer: Y, N
(C) INSUFFICIENT. We get a yes and no to two of our chosen values
Criteria: p = prime, p > 2, at least one term is divisible by p
Values: 3, 11
Answer: Y, N

How well did you know this?
1
Not at all
2
3
4
5
Perfectly
9
Q

Is x an integer?

(1) x/2 is an integer
(2) 2x is an integer

A

OG10, Lab # 2, Q 167. Answer A. The question does not require rephrasing. Apply the three step method for Yes/No testing (FIZ question).
(1) SUFFICIENT. We are not able to list a fraction, but we get two yes answers to integer and zero.
Criteria: x/2 = int
Values: 0, 2
Answer: Y, Y
(2) INSUFFICIENT. We get a yes and no to two of our chosen values
Criteria: 2x = int
Values: 1/2, 0, 1
Answer: N, Y

How well did you know this?
1
Not at all
2
3
4
5
Perfectly
10
Q

Is x^2 greater than x?

(1) x^2 is greater than 1
(2) x is greater than -1

A

OG10, Lab # 2, Q 183. Answer A. The question does not require rephrasing. Apply the three step method for Yes/No testing (Regular question).
(1) SUFFICIENT. We get yes answers for all five values.
Criteria: x^2 > 1
Values: 2, 3, 4, 5, 6
Answer: Y, Y, Y, Y, Y
(2) INSUFFICIENT. We get both yes and no for some of the values
Criteria: x > -1
Values: 0, 1, 2, 3, 4
Answer: N, N, Y

How well did you know this?
1
Not at all
2
3
4
5
Perfectly
11
Q

Is k > 0?

(1) 1/k > 0
(2) k^2 > 0

A

OG10, Lab # 2, Q 202. Answer A. The question does not require rephrasing. Apply the three step method for Yes/No testing (NPZ question).
(1) SUFFICIENT. We are only able to list one value for k, which says that it is greater than zero
Criteria: -10 < k < 10, 1/k >0
Values: 1
Answers: Y
(2) INSUFFICIENT. We are unable to list zero but get both yes and no for the other values
Criteria: -10 < k < 10, k^2 > 0
Values: -1, 1
Answers: N, Y

How well did you know this?
1
Not at all
2
3
4
5
Perfectly
12
Q

Is x between 0 and 1?

(1) x^2 is less than x
(2) x^3 is positive

A

OG10, Lab # 2, Q 206. Answer A. The question does not require rephrasing. Apply the three step method for Yes/No testing (FIZ question).
(1) SUFFICIENT. We are unable to list other possible integer or zero values for x
Criteria: x^2 < x
Values: 1/2
Answer: Y
(2) INSUFFICIENT. We are unable to list zero as a value, but we get yes and no for the other values
Criteria: x^3 > 0
Values: 1/2, 1
Answer: Y, N

How well did you know this?
1
Not at all
2
3
4
5
Perfectly
13
Q

If x is a positive integer, is Sqrt(x) an integer?

(1) Sqrt(4x) is an integer
(2) Sqrt(3x) is not an integer

A

OG10, Lab # 2, Q 222. Answer A. The question does not require rephrasing. Apply the three step method for Yes/No testing (Regular question).
(1) SUFFICIENT. We get all yes answers to our five values
Criteria: x > 0, x = integer, Sqrt(4x) = integer
Values: 1, 4, 9, 16, 25
Answer: Y, Y, Y, Y, Y
(2) INSUFFICIENT. We get both yes and no for some of our chosen values
Criteria: x > 0, x = integer, Sqrt(3x) is not an integer
Values: 1, 2, 4, 5, 6
Answer: Y, N

How well did you know this?
1
Not at all
2
3
4
5
Perfectly
14
Q

If 2xy + z = 9, what is the value of the positive integer z?

(1) xyz – z^2 = 0
(2) x + y – 3z = -5

A

CAT 1, Q 3. Answer A. Linear Equations. Since the question asks for the value of the positive integer z, it is a good idea to isolate z in the given equation. Rearranging (2xy + z = 9) yields (z = 9 – 2xy), so one rephrase of this question is “what is the value of xy?

(1) SUFFICIENT: This statement can be manipulated by factoring the variable z.
z(xy – z) = 0
This equation indicates that either z or the expression (xy – z) must equal zero. Given that z is a positive integer, it follows that:
xy – z = 0
xy = z

Note that this partially answers both the original question and the rephrased question; we know that z = 9 – 2xy, and that z = xy. By substituting z for xy in the original equation, we can solve for z:
2xy + z = 9
3z = 9
z = 3

(2) INSUFFICIENT: This equation cannot be manipulated or combined with the original equation to solve for any of the variables.

How well did you know this?
1
Not at all
2
3
4
5
Perfectly
15
Q

When the positive integer x is divided by 4, is the remainder equal to 3?

(1) When x is divided by 2, the remainder is 1.
(2) x is divisible by 3.

A

CAT 1, Q 6. Answer E. Divisibility & Primes.

(1) INSUFFICIENT: statement tells us that x is odd integer. Test 9/4 and remainder is 1
(2) INSUFFICIENT: statement tells us that x could be 3, 6, 9, 12…Tests 9/4 and remainder is 1
(1) and (2) INSUFFICIENT: x is an odd integer and multiple of 3. Test 9/4 and remainder is 1

How well did you know this?
1
Not at all
2
3
4
5
Perfectly
16
Q

Refer to picture (CAT 1, Q 7). Quadrilateral ABCD is a rhombus and points C, D, and E are on the same line. Is quadrilateral ABDE a rhombus?

(1) The measure of angle BCD is 60 degrees.
(2) AE is parallel to BD.

A

CAT 1, Q 7. Answer C. Polygons. (1) INSUFFICIENT: Draw segment BD. Since BC = CD (because ABCD is a rhombus), Triangle BCD is an isosceles triangle. Since angle BCD = 60, the remaining angles in the triangle must also equal 60 degrees and Triangle BCD is actually an equilateral triangle. Since BD must also be equal to AB and AD, Triangle ABD is also an equilateral triangle and all of its angles measure 60 degrees. Since CD is parallel to AB and DE is an extension of line CD, we know that DE is also parallel to AB. Using AD as a transversal, we know the measure of angle ADE = 60. However, we know nothing about angles DAE or AED and with only one pair of opposite sides parallel we cannot conclude that quadrilateral ABDE is a rhombus.

(2) INSUFFICIENT: Knowing that AE is parallel to BD allows us to conclude that alternate interior angles DAE and ADB are congruent. For the same reasons stated above, we know that DE is parallel to AB, and with two pairs of opposite sides parallel we know we have a parallelogram. But we have no further evidence that quadrilateral ABDE is a rhombus.
(1) and (2) SUFFICIENT: drawing segment BD creates three equilateral triangles. Already know that Quadrilateral ABDE is parallelogram, and since all sides are congruent, it is a rhombus

How well did you know this?
1
Not at all
2
3
4
5
Perfectly
17
Q

When the positive number a is rounded to the nearest tenth, the result is the number b. What is the tenths digit of a?

(1) When a is rounded to the nearest integer, the result is less than a.
(2) When b is rounded to the nearest integer, the result is greater than b.

A

CAT 1, Q 10. Answer C. Digits & Decimals.
(1) INSUFFICIENT. a must be rounded down to the nearest integer. Thus, the tenths digit could be 0, 1, 2, 3, 4. Because there are five possibilities for the tenths digit, there is no conclusion

(2) INSUFFICIENT. b must be rounded up to nearest integer. Thus, the tenths digit could be 5, 6, 7, 8, 9. Recall that the problem asks about a, not b; therefore, we need to figure out which possible values for a will then round to one of the tenths digits 5, 6, 7, 8, or 9. Lowest such value of a is xx.45 and highest such value of a is xx.9499. Thus, the tenths digit of a could be 4, 5, 6, 7, 8, 9. Because there are six possibilities for the tenths digit, there is no conclusion
(1) and (2) SUFFICIENT. The only common value between the two statements is 4. Thus, taking both statements together guarantees the tenths digit of a is 4.

How well did you know this?
1
Not at all
2
3
4
5
Perfectly
18
Q

What is the remainder when 25 is divided by positive integer j?

(1) j is even.
(2) j < 9

A

CAT 1, Q 13. Answer C. Divisibility & Primes.
(1) INSUFFICIENT. j could be 2, 4, 6, 8, 10…Test 25/10 (remainder 5) and 25/2 (remainder 1). Cannot conclude what the exact remainder is.

(2) INSUFFICIENT. j could be 1, 2, 3, 4, 5, 6, 7, 8, 9. Test 25/9 (remainder 7) and 25/2 (remainder 1). Cannot conclude what the exact remainder is.
(1) and (2) SUFFICIENT. J could be 2, 4, 6, 8. Test all values and remainder is always 1

How well did you know this?
1
Not at all
2
3
4
5
Perfectly
19
Q

Is x > 0?

(1) x^2 > 0
(2) x + 2 > 0

A

CAT 1, Q 14. Answer E. Inequalities.

(1) INSUFFICIENT. This tells us that x^2 is positive. If you square a positive number, you get a positive result. If you square a negative number, you also get a positive result. Therefore, we cannot tell from this information whether x is positive or negative
(2) INSUFFICIENT. This tells us that x + 2 > 0. If we subtract 2 from both sides, we get the following inequality: x > -2. According to this inequality, x could be positive (in which case it would definitely be greater than -2) or negative (x could be -1, for example).
(1) and (2). INSUFFICIENT. Both statements tell us that x could be positive or negative

How well did you know this?
1
Not at all
2
3
4
5
Perfectly
20
Q

A scientist is studying bacteria whose cell population doubles at constant intervals, at which times each cell in the population divides simultaneously. Four hours from now, immediately after the population doubles, the scientist will destroy the entire sample. How many cells will the population contain when the bacteria is destroyed?

(1) The population just divided and, since the population divided two hours ago, the population has quadrupled, increasing by 3,750 cells.
(2) The population will double to 40,000 cells with one hour remaining until the scientist destroys the sample.

A

CAT 1, Q 19. Answer A. Algebraic Translations. We need two additional pieces of information: (i) how frequently does the population double? (ii) what is the population size at any given time after it has doubled?

Let T = Now

(1) SUFFICIENT 
Pop(T) – Pop(T-2) = 3,750
Pop(T) = 4Pop(T-2) 
4Pop(T-2) – Pop(T-2) = 3,750 
Pop(T-2) = 1,250 (doubles six times between T-2 and T+4)
Pop(T) = 5,000 

Pop(T+4) = (2^6)(1,250) = 80,000

(2) INSUFFICIENT. this statement does not provide any information about how frequently the population is doubling

How well did you know this?
1
Not at all
2
3
4
5
Perfectly
21
Q

Is x a multiple of 4?

(1) x + 2 is divisible by 2
(2) 6 is a factor of 3x

A

CAT 1, Q 20. Answer E. Divisibility & Primes.
(1) INSUFFICIENT. if x + 2 is divisible by 2, then x itself must be divisible by 2, but not necessarily 4 (RULE: for x + y to be divisible by y, x itself must be divisible by y. x could be 2 (2 + 2 = 4, which is divisible by 2) and x could be 4 (4 + 2 = 6, which is divisible by 2). 4 is a multiple of 4 but 2 is not.

(2) INSUFFICIENT. x must be divisible by 2 because of the prime factorization of 6, but this does not guarantee that x is divisible by 2. For example, x = 2 results in 6, which is divisible by 6 but x is not a multiple of 4.
(1) and (2) INSUFFICIENT. Both statements combined lead to the same conclusion. x is even but not necessarily divisible by 4

How well did you know this?
1
Not at all
2
3
4
5
Perfectly
22
Q

Triangle BCD is inscribed in a circle. Angle BCD = x and angle BDC = y. If CD is the diameter of the circle, does x equal 30?

(1) The length of CD is twice the length of BD.
(2) y = 60

A

CAT 1, Q 22. Answer D. Triangles & Diagonals. Given CD = diameter, we know that angle CBD must be a right angle and CD is the hypotenuse

(1) SUFFICIENT. CD = 2BD means the ratio of BD to CD is 1:2. RULE: the side ratios of a 30-60-90 triangle are 1:1Sqrt(3):2. We can conclude that the triangle is a 30-60-90 triangle. Since BD is the short leg, the opposite angle (x) must be 30 degrees
(2) SUFFICIENT. Given that y = 60 degrees and that angle CBD is 90 degrees, we can conclude that x = 30 degrees

How well did you know this?
1
Not at all
2
3
4
5
Perfectly
23
Q

What is x?

(1) x2 + 3x + 2 = 0
(2) x < 0

A

CAT 1, Q 24. Answer E. Quadratic Equations.

(1) INSUFFICIENT. (x + 2)(x + 1) = 0. Thus, x equals either -2 OR -1. We do not have enough information to determine one specific value of x
(2) INSUFFICIENT. This statement says that x is negative, but we do not know the specific value
(1) and (2) INSUFFICIENT. do not provide enough information together to determine a specific value

How well did you know this?
1
Not at all
2
3
4
5
Perfectly
24
Q

Refer to picture (CAT 1, Q 28). Triangle ABC is inscribed inside a circle where, and line segment AB has length 18, what is the area of triangle ABC?

(1) Angle ABC measures 30 degrees
(2) The circumference of the circle is 18pi

A

CAT 1, Q 28. Answer C. Circles & Cylinders. Strategy is to prove that the triangle is a right triangle.

(1) INSUFFICIENT. Statement only tells us that angle ABC is 30 degrees, with no other information about the other angles. There is not enough information to determine the lengths of AC and BC
(2) INSUFFICIENT. C = 18pi. This statement tells us that AB = 18 = diameter. However, point C is still free to move around the circumference of the circle, giving different areas for the triangle.

(1) and (2). SUFFICIENT. With one side (AB) equal to the diameter, we know that angle ACB is 90. Statement (1) tells us that angle ABC is 30 degrees. Thus, we can conclude that angle BAC is 60. The triangle is a 30:60:90 triangle, and its sides have the ratio 1:1Sqrt(3):2.
Area = (1/2)(9)(9Sqrt(3)) = (81/2)Sqrt(3)

How well did you know this?
1
Not at all
2
3
4
5
Perfectly
25
Q

If n is not equal to 0, is |n| < 4 ?

(1) n2 > 16
(2) 1/|n| > n

A

CAT 1, Q 31. Answer A. Inequalities. Lines The statement can be rephrased to: (i) if n > 0, is n < 4? (ii) if n < 0, is n > -4? Combing the two questions yields: is -4 < n < 4?

(1) SUFFICIENT. We are able to conclude that n is definitely not between -4 and 4. Note that an absolute no is sufficient!
n^2 > 16
n > 4 OR n < -4

(2) INSUFFICIENT. Test 1 > |-4| * -4. 1 > -16. Thus, n can be any negative value since |n|*n will always be negative and therefore less than 1. This statement is insufficient because n might not be between -4 and 4.
1/|n| > n
1 > |n| * n

How well did you know this?
1
Not at all
2
3
4
5
Perfectly
26
Q

Three lines cross to form a triangle and external angles. Interior angle a is opposite to exterior angle e, interior angle g is adjacent to exterior angle c, and interior angle f is adjacent to angles d and b. What is the degree measure of angle a?

(1) b + c = 287 degrees
(2) d + e = 269 degrees

A

CAT 1, Q 33. Answer A. Lines & Angles. Note that a = e because they are vertical angles, and e + f + g = 180. We can rewrite this equation to be e = 180 – f – g. Thus, we are looking for f + g = e = a

(1) SUFFICIENT. This statement allows us to calculate f + g. More specifically:
b + c = 287
(b + f) + (c + g) = 180 + 180
287 + (f + g) = 360
(f + g) = 73 = e = a

(2) INSUFFICIENT. d + e = 269 OR d + a = 269. There are many combinations that satisfy this constrain. We are unable to determine a unique value for a

How well did you know this?
1
Not at all
2
3
4
5
Perfectly
27
Q

A certain panel is to be composed of exactly three women and exactly two men, chosen from x women and y men. How many different panels can be formed with these constraints?

(1) If two more women were available for selection, exactly 56 different groups of three women could be selected.
(2) x = y + 1

A

CAT 1, Q 34. Answer C. Combinatorics. 3 women + 2 men = panel. Chosen from x women and y men. In order to determine how many different panels that can be formed with these constraints, we need to know the values of x and y. The number of different panels that can be formed = xC3 * yC2
(1) INSUFFICIENT. Choosing 3 from x + 2 yields 56 groups. Thus, (x+2)C3 = 56. One concept that you need to know for the exam is that when dealing with combinations and permutations, each result corresponds to a unique set of circumstances. Because we know the number of groups yielded (in this case 56), then we know that there is only one possible of x (which is 6). However, we still do not know the value of y

(2) INSUFFICIENT. knowing that x = y + 1 is only helpful if we know one of the values. With only this information, there are infinite possible values for x and y.
(1) and (2). SUFFICIENT Together. We are able to determine x from statement (1), which allows us to calculate the value of y with statement (2)

NOTE: with data sufficiency problems, we do not need to actually be able to calculate exact values; it is enough to know that we CAN calculate them if we wanted to do so

How well did you know this?
1
Not at all
2
3
4
5
Perfectly
28
Q

The number of antelope in a certain herd increases every year by a constant factor. If there are 500 antelope in the herd today, how many years will it take for the number of antelope to double?

(1) Ten years from now, there will be more than ten times the current number of antelope in the herd.
(2) If the herd were to grow in number at twice its current rate, there would be 980 antelope in the group in two years.

A

CAT 1, Q 37. Answer B. Algebraic Translations.
Number of antelope in the herd today = 500
x = annual growth factor
1,000 2

We need to determine minimum n to yield a population of 1,000 or more. In order to solve this problem, we need to know the value of x.

(1) INSUFFICIENT. There are infinite values of x that make this inequality true
500x^10 > 5,000
x^10 > 10

(2) SUFFICIENT. Let y = the growth factor from the statement (Note that y does not necessarily equal 2x because x is a growth factor.
500y^2 = 980
y = Sqrt(980/500) = Sqrt(49/25) = 7/5 = 1.4
x = (1.4 – 1)/2 + 1 = 1.2

How well did you know this?
1
Not at all
2
3
4
5
Perfectly
29
Q

Does x fall between 20 and 26, inclusive?

(1) x is a multiple of 9
(2) x = 18

A

Lecture # 1. Answer D.

(1) SUFFICIENT. x can be 9, 18, 27, 36, etc. With this information, we can conclude that x does NOT fall between 20 and 26, inclusive, because multiples of 9 do not fall in this range
(2) SUFFICIENT. We are told that x is unequivocally equal to 18, which does not fall in the given range. Thus, we can conclude that x does NOT fall between 20 and 26, inclusive.

How well did you know this?
1
Not at all
2
3
4
5
Perfectly
30
Q

Is n an integer?

(1) n^2 is an integer
(2) Sqrt(n) is an integer

A

OG13, Lecture # 1, DS #172. Answer B.
(1) INSUFFICIENT. We can rephrase this question to say, “is the square root of an integer also an integer?” The answer is no, because if n is equal to the square of some prime number, then n^2 will be an integer, but n itself is not actually an integer. For example, if n = Sqrt(11), then n^2 = 11, which is an integer. However, n itself is not an integer. Thus, we do not have enough information to conclude that n is an integer

(2) SUFFICIENT. This can be rephrased to state that n = (integer)^2. For all values of n, Sqrt(n) is an integer AND n is an integer.

How well did you know this?
1
Not at all
2
3
4
5
Perfectly
31
Q

Is a – b + c > a + b – c?

(1) b is negative
(2) c is positive

A
Lecture # 1. Answer D. First we must detangle the complicated inequality in the question. 
a – b + c > a + b – c
- b + c > b – c
-2b > -2c
b < c 

(1) INSUFFICIENT. With this information, we know that b is negative, but the value of c could feasibly be even more negative than the value of b
(2) INSUFFICIENT. With this information, we know that c is positive, but the value of b could feasibly be even more positive than the value of c
(1) and (2) SUFFICIENT. If b is negative and c is positive, we can conclude that the original inequality is true because a negative will always be less than a positive

How well did you know this?
1
Not at all
2
3
4
5
Perfectly
32
Q

If k is an integer and p is a factor of k, then is 1 < p < k?

(1) k > 4!
(2) 13! + 2 is less than or equal to k, which is less than or equal to 13! + 13

A

OG11, Lecture # 1, DS # 153. Answer B. The question can be rephrased to state, “is k prime?” because only prime numbers only have factors of 1 and themselves.

(1) INSUFFICIENT. k > 432*1 or k > 24. k could be prime (e.g. 31), in which case the inequality in the question does not hold true. k could also not be prime (e.g. 27), in which case the inequality does hold true.
(2) SUFFICIENT. None of the numbers that fall between 13! + 2 and 13! + 13 are prime (to test this, draw a number line). As a result, we know that the inequality in the question holds true.

How well did you know this?
1
Not at all
2
3
4
5
Perfectly
33
Q

Company X has exactly two product lines and no other sources of revenue. If the consumer products line experiences a k% increase in revenue (where k is a positive integer) in 2010 from 2009 levels and the machine parts line experiences k% decrease in revenue in 2010 from 2009 levels, did Company X’s overall revenue increase or decrease in 2010?

(1) In 2009, the consumer products line generated more revenue than the machine parts line
(2) k = 8

A

FDP Strategy, Ch 6, Q 7. Answer A. Total Revenue in 2009 = C + M. Total Revenue in 2010 = (1+k)C + (1-k)M

Since you already know that k is a positive integer, you know that C increases by the same percent by which M decreases. So, you do not actually need to know k. You already know that k percent of whichever number is bigger (C or M) will constitute a bigger change to the overall revenue. If C starts off bigger, then a k% increase in C means more new dollars coming in than you would lose due to k% decrease in the smaller number, M.

The question can thus be rephrased, “Which is bigger, C or M?”

(1) SUFFICIENT. this statement tells you that C is bigger than M. Thus, k% increase in c is larger than k% decrease in M, and the overall revenue went up
(2) INSUFFICIENT. Knwoing the percent change doesn’t help, since you do not know whether C or M is bigger

How well did you know this?
1
Not at all
2
3
4
5
Perfectly
34
Q

What is the ratio of x:y:z?

(1) x + y = 2z
(2) 2x + 3y = z

A

FDP Strategy, Ch 6, Q 9. Answer C. For this problem, you do not necessarily need to know the value of x, y or z. You simply need to know the ratio of x:y:z (in other words, the value of x/y AND y/z). You must manipulate the information given to see whether you can determine this ratio

(1) INSUFFICIENT. There is no way to manipulate this equation to solve for a ratio. 
x + y = 2z
x = 2z – y 
x/y = (2z – y)/y = 2z/y – 1 
(2) INSUFFICIENT. Same as above
2x + 3y = z
2x = z – 3y 
x/y = (z – 3y)/2y = z/2y – 3/2
(1) and (2) SUFFICIENT. Use substitution to combine the equations
x + y = 2(2x + 3y)
x/y = -5/3

2(2z – y) + 3y = z
y/z = -3

Therefore, the ratio x:y:z = 5:-3:1

How well did you know this?
1
Not at all
2
3
4
5
Perfectly
35
Q

A certain freeway has Exits J, K, L and M in that order. What is the road distance from Exit K to Exit L?

(1) The road distance from Exit J to Exit L is 21km.
(2) The road distance from Exit K Exit M is 26km.

A

OG13, HW1, Q6. Word Problem. Answer E. In this Number Line question, we have to figure out something about the relationship between K and L

(1) INSUFFICIENT. Does not provide enough information to calculate the distance between K and L
(2) INSUFFICIENT. Does not provide enough information to calculate the distance between K and L
(1) and (2) INSUFFICIENT. Even with both statements, there are multiple possible values for the distance from K to L. In essence, the fixed distances (21km and 26km) can slide past each other, changing the K-to-L distance. Below are two examples
JK = 11, KL = 10, LM =16
JK = 16, KL = 5, LM =21

How well did you know this?
1
Not at all
2
3
4
5
Perfectly
36
Q

If n + k = m, what is the value of k?

(1) n = 10
(2) m + 10 = n

A

OG13, HW1, Q17. Linear Equations. Answer B. We are trying to find the value of, k = m – n. It would be sufficient to know either k or m – n

(1) INSUFFICIENT. k = m – 10. There are an infinite number of values for m, thus changing the value of k.
(2) SUFFICIENT. m + 10 = n can be rewritten to say m – n = -10, which is the value of k

How well did you know this?
1
Not at all
2
3
4
5
Perfectly
37
Q

The selling price of an article is equal to the cost of the article plus the markup. The markup on a certain television set is what percent of the selling price?

(1) The markup on the television set is 25 percent of the cost.
(2) The selling price of the television set is $250.

A

OG13, HW1, Q40. FDP. Answer A. Let S = the selling price of the TV, C = the cost of the TV, and M = markup of the TV. Thus, we can write the following equation: S = C + M. We are trying to determine M/S.

(1) SUFFICIENT. Using substitution, we can determine M/S. We are told that M = 0.25C, which can be substituted into the original equation:
S = C + 0.25C
S = 1.25C
M/S = 0.25C/1.25C = 1/5
(2) INSUFFICIENT. This statement tells us that the sum of the cost and markup is $250, but the cost and markup could be any two numbers summing to that amount. Thus, the markup could be anywhere from 0% to 100% of the cost. The markup-price ratio is not fixed.

How well did you know this?
1
Not at all
2
3
4
5
Perfectly
38
Q

If m is an integer, is m odd?

(1) m/2 is not an even integer.
(2) m – 3 is an even integer

A

OG13, HW1, Q72. Number Properties. Answer B. If m is an integer, is m odd? No rephrasing is necessary

(1) INSUFFICIENT. m/2 is NOT an even integer. This does NOT mean that m/2 is an odd integer! Rather, it may be that m/2 is not an integer at all. Set up a table to test scenarios. Since both examples conform to the condition given in statement 1 yet give different answers to the question, the statement is not sufficient.
m m/2 = not even m = odd int?
1 1/2 Yes
2 1 No
(2) SUFFICIENT. Apply the properties of odds and evens. If m – 3 = even integer, then m = even integer + 3. m = even + odd is always odd!

How well did you know this?
1
Not at all
2
3
4
5
Perfectly
39
Q
Each of the letters in the table below represents one of the numbers 1, 2, 3, and each of these numbers occurs exactly once in each row and exactly once in each column. What is the value of r?
r  s  t
u v w
x y  z
(1) v + z = 6
(2) s + t + u + x = 6
A

OG13, HW1, Q103. Word Problems. Answer D. This is Sudoku! This means each digit must occur exactly once in each column and each row. This also means that there will be only three 1’s, three 2’s and three 3’s. We are asked for the value of r.

(1) SUFFICIENT. If v + z = 6, then both v and z must equal 3, since 3 + 3 is the only way to achieve 6 if only 1’s, 2’s and 3’s can be used. If v and z are 3, then we know the other numbers sharing either a row or a column with v or z cannot be 3. The only remaining possibility is r = 3.
(2) SUFFICIENT. We know that r = 6 – s – t and that r = 6 – u – x. Using substitution:
6 – s – t = 6 – u – x
s + t = u + x

Using the information given in statement 2 and substituting:
s + t = 6 – u – x
6 – u – x = u + x
6 = 2u + 2x
The only possibility is: u = 2 or 1, x = 2 or 1. Thus, r must be equal to 3

How well did you know this?
1
Not at all
2
3
4
5
Perfectly
40
Q

If x and y are integers, is xy even?

(1) x = y + 1
(2) x/y is an even integer

A

OG13, HW1, Q111. Number Properties. Answer D. In this odds and evens problem, we are asked whether the product of xy is even. In order for this to be true, just one of the two integers needs to be even. By the properties of odds and evens, an even integer multiplied by any other integer produces an even product. Therefore, we can rephrase the question as “Is x OR y even?”

(1) SUFFICIENT. If x = y + 1, then we are dealing with two consecutive integers. In any pair of consecutive integers, one of the integers must be even and one must be odd. The answer to the rephrased question is yes.
(2) SUFFICIENT. If x/y is an even integer, then x must be even because an odd number contains only odd prime factors. Thus, if x were odd and divided by y, the quotient would either be an odd integer or a fraction. Since x is definitely even, the answer to the rephrased question is yes.

How well did you know this?
1
Not at all
2
3
4
5
Perfectly
41
Q

Six shipments of machine parts were shipped from a factory on two trucks, with each shipment entirely on one of the trucks. Each shipment was labeled either S1, S2, S3, S4, S5, S6. The table below shows the value of each shipment as a fraction of the total value of the six shipments. If the shipments on the first truck had a value greater than 1/2 of the total value of the six shipments, was S3 shipped on the first truck?
Shipment: S1 = 1/4, S2 = 1/5, S3 = 1/6, S4 = 3/20, S5 = 2/15, S6 = 1/10
(1) S2 and S4 were shipped on the first truck.
(2) S1 and S6 were shipped on the second truck.

A

OG13, HW1, Q131. FDP. Answer B. Let S = total value of all six shipments, Truck1 = value of shipments on truck 1 and Truck2 = value of shipments on truck 2. Given that Truck1 > 1/2S, we know that Truck2

How well did you know this?
1
Not at all
2
3
4
5
Perfectly
42
Q

Last year, a certain company began manufacturing product X and sold every unit of product X that it produced. Last year the company’s total expenses for manufacturing product X were equal to $100,000 plus 5 percent of the company’s total revenue from all units of product X sold. If the company made a profit on product X last year, did the company sell more than 21,000 units of product X last year?

(1) The company’s total revenue from the sale of product X last year was greater than $110,000.
(2) For each unit of product X sold last year, the company’s revenue was $5.

A

OG13, HW1, Q141. Word Problem (Algebraic Translation). Answer B. Translate the words into equations
Profit = PQ – 100,000 – 0.05(PQ) = 0.95(PQ) – 100,000 = (19/20)(PQ) – 100,000
Profit > 0
(19/20)(PQ) – 100,000 > 0
R = P
Q
Is Q > 21,000? We need to know price in order to determine Q.

(1) INSUFFICIENT. R > 110,000. Without knowing the price per unit, it will be impossible to tell how many units were sold.

(2) SUFFICIENT. Incorporating P = 5 into the equality allows us to solve for Q. After doing the first few steps of long division, we are able to determine that Q > 21,000
(19/20)(5Q) > 100,000
Q > 100,000/(19/4)
Q > 400,000/19

How well did you know this?
1
Not at all
2
3
4
5
Perfectly
43
Q

Are all of the numbers in a certain list of 15 numbers equal?

(1) The sum of all the numbers in the list is 60.
(2) The sum of any 3 numbers in the list is 12.

A

OG13, HW1, Q144. Word Problem. Answer B. The question cannot be usefully rephrased.

(1) INSUFFICIENT. The sum of 15 numbers is 60. Each of the numbers could be 4, but there are many other possibilities.
(2) SUFFICIENT. The sum of ANY 3 numbers is 12. At first glance, it appears that each number must equal 4. In order to prove this fact, test scenarios. Make a list of numbers of which any 3 numbers sum to 12. It will quickly become apparent that this is impossible unless each number is equal to 4.
4,4,4,4,4,4,4,4,4,4,4,4,4,5,3 = 60
Example 1: 4 + 5 + 3 = 12
Example 2: 4 + 4 + 5 = 13

How well did you know this?
1
Not at all
2
3
4
5
Perfectly
44
Q

The number A is a two-digit positive integer; the number B is the same two-digit positive integer formed by reversing the digits of A. If Q = 10B – A, what is the value of Q?

(1) The tens digit of A is 7.
(2) The tens digit of B is 6.

A

FDP Strategy Guide, Ch 7, Q6. FDP. Answer B. We can write two equations with the given information where x is the tens digit and y is the units digit for A. We can use these equations to rewrite the given equation for Q and determine that Q is only dependent on y. Thus, we can rephrase the question as: “what is the value of the units digit of A or the tens unit of B?”
A = 10x + y
B = 10y + x
Q = 10(10y + x) – (10x + y) = 100y + 10x – 10x – y = 99y
(1) INSUFFICIENT. Tens digit of A is 7 means that x = 7. This is insufficient to provide the value of Q.
(2) SUFFICIENT. Tens digit of B is 6 means that y = 6. Thus Q = 99(6)

How well did you know this?
1
Not at all
2
3
4
5
Perfectly
45
Q

If n is a positive integer, is (1/10)^n < 0.01?

(1) n > 2
(2) (1/10)^(n-1) < 0.1

A
OG13, Lecture # 2, Q169. Exponents & Roots. Answer D. We can rephrase the question by manipulating the inequality
(1/10)^n < (1/100)
(1/10)^n < 1/(10^2)
1/(10^n) < 1/(10^2)
n > 2?

(1) SUFFICIENT. The statement tells us that n is greater than 2, which answers our rephrased question.
(2) SUFFICIENT. We need to rephrase this as well.
(1/10)^(n-1) < 1/10
1/(10^(n-1)) < 1/10
n – 1 > 1
n > 2

How well did you know this?
1
Not at all
2
3
4
5
Perfectly
46
Q

If x and y are nonnegative integers and x + y = 25, what is x?

(1) 20x + 10y < 300
(2) 20x + 10y > 280

A

Algebra Strategy Guide, Ch 9.
(1) INSUFFICIENT. Does not tell you enough to solve for x
(2) INSUFFICIENT. Does not tell you enough to solve for x
(1) and (2) SUFFICIENT. Substitute y = 25 – x. Since x must be an integer, x must equal 4
280 < 20x + 10y < 300
280 < 20x + 10(25 – x) < 300
280 < 20x + 250 – 10x < 300
30 < 10x < 50
3 < x < 5

47
Q

Guy’s net income equals his gross income minus his deductions. By what percent did Guy’s net income change on January 1, 1989, when both his gross income and his deductions increased?

(1) Guy’s gross income increased by 4% on January 1, 1989
(2) Guy’s deductions increased by 15% on January 1, 1989

A

OG13, HW2, Q55.
Percent Change = $ Change in Net Income / $ Original Net Income
Net Income = Gross Income – Deductions

(1) INSUFFICIENT. Guy’s gross income increased by 4% on the specific date in question. This does not allow us to calculated either the change in net income or the starting net income.
(2) INSUFFICIENT. Guy’s deductions increased by 15% on the specific date in question. This does not allow us to calculated either the change in net income or the starting net income.
(C) INSUFFICIENT. We know that gross income increases by 4% and that deductions increased by 15%, but we do not know the starting or ending values of net income. Thus, we cannot calculate the change or original. To prove insufficiency, develop contradictory examples.
Example 1:
Gross Income = 100 – 104
Deductions = 10 – 11.5
Net Income = 90 – 92.5
% Change = 2.5 / 90 = 2.78%

Example 2: 
Gross Income = 50 -- 52
Deductions = 30 – 34.50
Net Income = 20 – 17.5
% Change = -2.5 / 20 = -12.5%
48
Q

If a, b, c and d are positive numbers, is a/b < c/d?

1) 0 < (c – a) / (d – b
(2) (ad/bc)^2 < ad/bc

A

OG13, HW2, Q92. We can rewrite the question to be ad < bc? We usually cannot cross multiply variables across an inequality because these variables could be negative. However, we are told that they are positive.

(2) SUFFICIENT. (ad/bc)^2 < ad/bc is true of positive fractions. Thus, 0 < ad/bc < 1 and 0 < ad < bc

(1) INSUFFICIENT. 0 < (c – a) / (d – b). In order for this to be true, both the numerator and denominator must be negative or positive.
- If both are positive, then c > a and d > b
- If both are negative, then a > c and b > d

In order to prove insufficiency, use real numbers.

(4) (2) < (1)(1) NO
(1) (1) < (4)(2) YES

49
Q

A department manager distributed a number of pens, pencils, and pads among the staff in the department, with each staff member receiving x pens, y pencils, and z pads. How many staff members were in the department?

(1) The number of pens, pencils, and pads that each staff member received were in the ratio 2:3:4, respectively.
(2) The manager distributed a total of 18 pens, 27 pencils, and 36 pads.

A

OG13, HW2, Q114. All of the numbers have to be integers and the number of employees must evenly divide each of x, y and z.
(1) INSUFFICIENT. Even with w as an unknown multiplier, we do not know the number of employees
x = 2w
y = 3w
z = 4w

(2) INSUFFICIENT. 18 pens, 27 pencils and 36 pads. All of these numbers are divisible by 9. Assuming the items are distributed evenly, there could be 9, 3, or 1 employees.

(C) INSUFFICIENT. 2:3:4 is the same as 6:9:12, which is the same as 18:27:36. Statement 1 provides no new information beyond what statement 2 has already given us

50
Q

In the expression below, if xn cannot equal 0, what is the value of S?
S = (2/n) / [(1/x) + (2/3x)]
(1) x = 2n
(2) n = 1/2

A
OG13, Lecture # 3, Q 156. Linear Equations. Answer A. Rephrasing the equation yield the question: “what is x/n?”
S = (2/n) / [(1/x) + (2/3x)]
S = (2/n) / [(3/3x) + (2/3x)]
S = (2/n) / (5/3x) 
S = 6x / 5n ? 

(1) SUFFICIENT. x = 2n can be substituted into the equation to get 6(2n) / 5n = 12/5
(2) INSUFFICIENT. n = 1/2 can be substituted into the equation but we still do not know the value of x

51
Q

What is the value of x^2 – y^2?

1) x – y = y + 2
(2) x – y = 1 / (x + y

A

Quant2, Lecture #3, Q 46. Quadratic Equations. Answer B. We can rephrase the question to: x^2 – y^2 = (x – y)(x + y) = ?

(2) SUFFICIENT. This can easily be rephrased to give us one value for the expression
x – y = 1/(x + y)
(x – y)(x + y) = 1

(1) INSUFFICIENT. x – y = y + 2. We can substitute this into the question but this will not help us.

52
Q

If x^2 + y^2 = 29, what is the value of (x – y)^2?

(1) xy = 10
(2) x = 5

A
Quant2, Lecture # 3, Q 73. Quadratic Equations. Answer A. We can rephrase the question using the equation given in the question stem:
x^2 – 2xy + y^2 = ?
x^2 + y^2 – 2xy = ?
29 – 2xy = ?
What is the value of xy?

(1) SUFFICIENT. This gives us exactly what we need
(2) INSUFFICIENT. x = 5 does not tell us the value of xy

Takeaway: immediately use the information that is given to you in the question.

53
Q

Is rst = 1?

(1) rs = 1
(2) st = 1

A

OG13, HW 3, Q 98. Linear Equations. Answer E. There is no rephrasing to be done for this linear equations question.
(1) INSUFFICIENT. If rs = 1, then r could be anything
(2) INSUFFICIENT. If st = 1, then r could be anything
(C) INSUFFICIENT. We can combine the equations, trying to put rst on one side of the resulting equation. We cannot isolate rst on one side of the equation and have a number by itself on the other side. Therefore, we cannot find a specific value for rst, and we cannot confirm or deny that rst = 1
(rs)(st) = (1)(1)
r * t * s^2 = 1
rst = 1/2

54
Q

What is the value of integer n?

(1) n(n + 1) = 6
(2) 2^2n = 16

A
OG13, HW 3, Q 67. 
(1) INSUFFICIENT. We can solve the factored quadratic equation, which has two solutions.
n(n + 1) = 6
n^2 + n – 6 = 0
(n + 3)(n – 2) = 0
n = -3 OR n = 2
(2) SUFFICIENT. We can simplify the equation and solve for one value of n
2^2n = 16
(2^2)^n = 16
4^n = 4^2
n = 2
55
Q

What is the value of ǀxǀ?

(1) x = -ǀxǀ
(2) x^2 = 4

A

OG13, HW 3, Q 1. Linear Equations. Answer B. No need to rephrase the question

(2) SUFFICIENT. If x^2 =4, then x must equal either 2 or – 2. Since ǀ2ǀ = 2 and ǀ-2ǀ = 2, it must be true that ǀxǀ = 2
(1) INSUFFICIENT. If x = -ǀxǀ, then we know that x must be negative or zero, since it equals the negation of an absolute value, which will always be positive or zero. However, this does NOT tell us anything about the specific numeric value of ǀxǀ. For example, ǀxǀ could equal 5 or 7, etc.

Takeaways:

  • If you know a variable’s Square, you know its absolute value
  • the expression -ǀxǀ means that x must be negative or zero
56
Q

If r and s are the roots of the equation x^2 +bx + c = 0, where b and c are constants, is rs < 0?

(1) b < 0
(2) c < 0

A

OG13, HW 3, Q99. Quadratic Equations. Answer B. The roots of a quadratic equation are the solutions to that equation, so this equation tells us that:
(x – r)(x – s) = x^2 + bx + c = 0
x^2 – sx – rx + rs = x^2 + bx + c = 0
x^2 – x(s + r) + rs = x^2 + bx + c = 0

So we know that:
b = -(s + r)
c = rs

We can rephrase the question to ask: is c < 0?
(2) SUFFICIENT. We know that either r is negative and s is positive or vice versa. Either way, rs would have to be less than 0
(1) INSUFFICIENT. If b is negative, we know that –(r + s) is negative, so r + s must be positive. Test numbers to prove insufficiency to the yes/no question.
-(r + s) < 0
r + s > 0

57
Q

If n is a positive integer, is the value of b – a at least twice the value of 3^n – 2^n?

(1) a = 2^(n+1) and b = 3^(n+1)
(2) n = 3

A

OG 13, HW 3, Q 160. Exponents & Roots. Answer A. We can rephrase the question, translating from words to symbols.
Is b – a ≥ 2(3^n – 2^n)?
Is b – a ≥ 2(3^n) – 2(2^n)

(2) INSUFFICIENT. This statement does not tell us anything about a or b.
(1) SUFFICIENT. We can manipulate the information in this statement to get b – a on one side of an equation. To do so, subtract equations. The tricky step is recognizing that we have to rewrite 3^(n+1) as (3^n)(3), which applies a subtle exponents rule.

Is b – a ≥ 2(3^n) – 2(2^n)?
Is 3(3^n) – 2(2^n) ≥ 2(3^n) – 2(2^n)?
Is 3(3^n) ≥ 2(3^n)?
Is 3 ≥ 2?

The answer to this question will always be Yes. Thus, this statement is sufficient.

58
Q

Every member of a certain club volunteers to contribute equally to the purchase of a $60 gift certificate. How many members does the club have?

(1) Each member’s contribution is to be $4.
(2) If 5 club members fail to contribute, the share of each contributing member will increase by $2.

A

OG13, HW 3, Q 163. Quadratic Equations. Answer D. Let M = # of members and C = dollar contribution per person. Thus, 60 = M * C or M = 60/C. The question can be rephrased to “what is 60/C?” or simply “what is C?”
(1) SUFFICIENT. If C = 4, then M = 60/4 = 15
(2) SUFFICIENT. We have two equations and two variables, but we still need to solve to make sure there we can come to one value. From the question stem, we know that C = 60/M, which we can substitute into our new equation from statement 2.
60 = (m – 5)(c + 2)
60 = (m – 5)(60/m + 2)
60 = 60 + 2m – 300/m – 10
0 = -10 + 2m – 300/m
0 = 2m^2 – 10m – 300
0 = m^2 – 5m – 150
0 = (m – 15)(m + 10)
m = 15 OR m = -10

Since the number of club members must be positive, the only valid solution is m = 15.

BEWARE of assuming that a quadratic will always produce two solutions and thus fail to provide a sufficient answer for a value question. On difficult GMAT questions, one of the solutions may be either invalid or redundant, leaving only one (therefore sufficient) answer. The safest policy is to factor and solve any quadratic before answering the original question.

59
Q

What is the value of (2t + t – x)/(t – x)?

(1) 2t/(t – x) = 3
(2) t – x = 5

A

OG13, HW 3, Q 171. Linear Equations. Answer A. The common term of t – x in both the numerator and denominator allows us to do strategic direct algebra:
(2t + t – x)/(t – x) = [(2t) + (t – x)] / (t – x) = 2t/(t – x) + (t – x)/(t – x) = 2t/(t – x) + 1

The problem hinted at this manipulation by writing the numerator as 2t + t – x, rather than as the more normal form 3t – x. The only value that is still unknown is 2t/(t – x). The question can be rephrased to ask, “what is the value of 2t/(t – x)”?

(1) SUFFICIENT. This answers the rephrased question
(2) INSUFFICIENT. We cannot manipulate t – x = 5 to determine the value of 2t/(t – x)

60
Q

For any integers x and y, min(x, y) and max(x, y) denote the minimum and maximum of x and y, respectively. For example, min(5, 2) = 2 and max(5, 2) = 5. For the integer w, what is the value of min(10, w)?

(1) w = max(20, z) for some integer z
(2) w = max(10, w)

A

OG13, Lecture # 4, Q 118. Formulas. Answer D. This problem is asking us to figure out whether w or 10 is smaller. If w is greater than 10, then min(10, w) = 10. If w is less than 10, min(10, w) = w.
(2) SUFFICIENT. w = max(10, w) means that w equals 10 or something greater than 10. In other words, w ≥ 10. Thus, we can substitute this value into the question. Min(10, 10 or something greater than 10) = 10
(1) SUFFICIENT. To deal with this statement, we need to think about possible values of z. Specifically, we need to consider values of z that are less than and greater than 20 (because that will change the value of max(20, z)
If z = 15, then max(20, z) = 20
If z = 25, then max(20, z = 25
No matter what value we pick for z, the value of w will have to be at least 20 (i.e. 20 or something greater than 20), which means the value of min(10, w) = 10

61
Q

If Carmen had 12 more tapes, she would have twice as many tapes as Rafael. Does Carmen have fewer tapes than Rafael?

(1) Rafael has more than 5 tapes
(2) Carmen has fewer than 12 tapes

A

OG13, Lecture # 5, Q 158. Algebraic Translations. Answer B. Assign C to the number of tapes that Carmen has and R to the number of tapes that Rafael has. The question stem states that C + 12 = 2R, which can be rewritten to say C = 2R – 12. We are asked if C < R, which can be rephrased to: 2R – 12 < R?; R < 12? Alternatively, we could simplify the expression in terms of C. If C = 2R – 12, then R = (C + 12)/2 and the inequality becomes: C < (C + 12)/2?; C < 12?

We have two rephrasings for this question, either of which would be sufficient: “Is R < 12 OR is C < 12?”

(1) INSUFFICIENT. R could be either greater than or less than 12
(2) SUFFICIENT. C is less than 12. This is a direct answer to the second rephrasing of the original question.

ALTERNATIVE METHOD: Test numbers. With inequalities, we only care about the edges so test the limits!

(1) INSUFFICIENT. R = 6+
Low end: R = 6, which means C = 0. Thus, C < R
High end: R = 100, which means C = 188. Thus, C > R
(2) SUFFICIENT. C = 11-
High end: C = 10, which means R = 11. Thus, C < R
Low end: C = 0, which means R = 6. Thus, C < R

Takeaways:

  • For inequality questions, TEST THE LIMITS!
  • Beware of hidden constraints! (e.g. positive integer questions)
62
Q

Tom, Jane, and Sue each purchased a new house. The average (arithmetic mean) price of the three houses was $120,000. What was the median price of the three houses?

(1) The price of Tom’s house $110,000
(2) The price of Jane’s house was $120,000

A

OG13, Lecture # 5, Q 110. Statistics. Answer B. We are told 120,000 = (T + J + S) / 3. Thus, the sum of T, J, and S is $360,000. We must find the median of T, J, and S.

(1) INSUFFICIENT. Knowing that Tom’s house cost $110,000 tells us that Tom’s house is not the most expensive house because it is lower than the average and that J + S = $250. However, we do not know whether Tom’s house was the least expensive or the middle house. Test scenarios. If Tom’s house is the least expensive, then the middle house would cost more than $110,000, and the median would be some number greater than $110,000. If Tom’s house is the middle house, however, then the median would be $110,000
(2) SUFFICIENT. Test scenarios. Jane’s house cannot be less expensive than both of the others, or else the average would have been higher than $120,000. By the same token, we know that Jane’s house cannot be more expensive than bother of the others. Otherwise, the overall average would have to be less than $120,000. There are only two viable scenarios: (1) All three houses are worth different amounts and Jane’s house is in the middle (2) All three houses are worth $120,000. In either case, the median is $120,000

Takeaways:
-Given average and one number equal to the average, we know that the median is equal to that number

63
Q

If a < x < b and c < y < d, is x < y?

(1) a < c
(2) b < c

A

OG13, HW 4, Q 48. Inequalities. Answer B.
(1) If a < c, we know that the lower range of possibilities for x is smaller than the lower range for y. Using only this information, we are unable to say for certain that x itself is smaller than y. We can test numbers to verify our thinking. For example, suppose a = 0 and c = 2.
If x = 2 and y = 3, then x < y
If x = 4 and y = 3, then x > y
(2) SUFFICIENT. If b < c, the upper limit for x is smaller than the lower limit for y. What this means is that the biggest x can be is smaller than the smallest y can be. Therefore, x must be smaller than y

64
Q

If x and y are positive, is x < 10 < y?

(1) x < y and xy = 100
(2) x^2 < 100 < y^2

A

OG13, HW 4, Q 52. Inequalities. Answer D. The question asks whether x < 10 < y

(1) SUFFICIENT. Since xy = 100 and both x and y are positive, either x = y = 10, or one of the variables is greater than 10 and the other one is less than 10. Since this assessment also tells us that x < y, we know that x < 10 and y > 10
(2) SUFFICIENT. Since we know that x and y are both positive, we can square root this inequality, which tells us that x < 10 < y

65
Q

If ǀxǀ denotes the least integer greater than or equal to x, is ǀxǀ = 0?

(1) -1 < x < 1
(2) x < 0

A

OG13, HW 4, Q 96. Formulas. Answer C. The question asks whether 0 is the smallest integer that is greater than or equal to x. For this to be true, x would have to be less than or equal to 0 but greater than -1. The question can be rephrased as “Is -1 < x ≤ 0?”
(1) INSUFFICIENT. If x = -0.5, then the smallest integer that is greater than x is 0. However, if x = 0.5, then the smallest integer that is greater than x is 1.
(2) INSUFFICIENT. There is no way to solve for an exact x here, so a good approach is to try to prove insufficiency by testing numbers. We know that if x = -0.5, then the smallest integer that is greater than x is 0. However, if x = -3.5, then the smallest integer that is greater than x is -3.
(C) SUFFICIENT. The combined statements tell us that -1 < x < 0. The smallest integer that is greater than every number that is greater than every number in that range is 0.

66
Q

Each gift certificate sold yesterday by a certain bookstore cost either $10 or $50. If yesterday the bookstore sold more than 5 gift certificates that cost $50 each, what was the total number of gift certificates sold yesterday by the bookstore?

(1) Yesterday the bookstore sold fewer than 10 gift certificates that cost $10 each
(2) The total cost of gift certificates sold yesterday by the bookstore was $460

A

OG13, HW 5, Q 57. Answer E. Algebraic Translations. Let T equal the number of $10 gift certificates sold and F for the number of $50 gift certificates sold. The given information can be translated and the question can be rephrased as “T + F = ?”
F > 5
T and F are both integers
(1) INSUFFICIENT. We are told T 5, this is not enough information to tell use the value of T + F
(2) INSUFFICIENT. We are told 10T + 50F = 460. If this were a simple two variable, linear equation, it would not be solvable. However, the integer constraints and F > 5 greatly reduces the number of possible solutions. We know that F must be at least 6, we know that 6 * 50 = $300 worth of $50 gift certificates were sold, leaving $160 worth of gift certificates to account for. However, $160 worth of gift certificates could be formed a few different ways, which means there is more than one possible answer to the question:
3(50) + 1(10) = $160
2(50) + 6(10) = $160
(C) INSUFFICIENT. Even if both statements are true, more than one example from our statement 2 analysis matches the information. There could be three $50 and one $10 or two $50 and six $10.

67
Q

Three machines, K, M and P, working simultaneously and independently at their respective constant rates, can complete a certain task in 24 minutes. How long does it take Machine K, working alone at its constant rate, to complete the task?

(1) Machines M and P, working simultaneously and independently at their respective constant rates, can complete the task in 36 minutes.
(2) Machines K and P, working simultaneously and independently at their respective constant rates, can complete the task in 48 minutes.

A

OG13, HW 5, Q 68. Answer A. Rates & Work. Set up a RTW chart for the three machines and the combination of all three machines working together. The question asks for the time it takes Machine K to complete the task. We would be able to solve for K’s time if we any of the following (a) K’s rate (2) M + P combined rate (b) M and P combined rate (c) M and P combined time.

(1) SUFFICIENT. We are given the time it takes M and P to complete the task together (36 minutes). We know that k + m + p = Total Rate = 1/24. Thus, k + (1/36) = 1/24, which allows us to solve for K’s rate and K’s time.
(2) INSUFFICIENT. This statement provides the time necessary for K and P to complete the task when working together. This time could be used to find the combined rate of K and P. However, we cannot determine the individual rate of K.

Takeaways:
-GMAT often gives you the relationship among three variables (k + m + p) and asks us for the value of one of the variables; the right combo of variables will do the trick!

68
Q

Material A costs $3 per kilogram, and Materials B costs $5 per kilogram. If 10 kilograms of Material K consists of x kilograms of Material A and y kilograms of Material B, is x > y?

(1) y > 4
(2) The cost of the 10 kilograms of Material K is less than $40.

A

OG13, HW 5, Q 105. Answer B. Statistics. We are told that X + Y = 10 and 3X + 5Y = Cost. We are asked if X > Y. Since X + Y = 10, we can rephrase the question is X > 10 – X OR is X > 5? Alternatively, the question can be phrased is Y 4, which means Y could equal 5, in which case X = Y. Or Y could equal 6, in which case Y > X. Or Y could equal 4.1, in which case Y 5 and X > Y

Takeaways:
-In weighted average problems, think about a see-saw

69
Q

While on a straight road, Car X and Car Y are traveling at different constant rates. If Car X is now 1 mile ahead of Car Y, how many minutes from now will Car X be 2 miles ahead of Car Y?

(1) Car X is traveling at 50 miles per hour and Car Y is traveling at 40 miles per hour
(2) Three minutes ago Car X was 1/2 mile ahead of Car Y

A

OG13, HW 5, Q 106. Answer D. Rates & Work. Draw a picture of the cars both NOW and LATER to ensure that you can visualize the problem. Use relative rates; T = Difference in Distance between NOW and LATER / Difference between the Rates. We need to find the difference in rates between X and Y.

(1) SUFFICIENT. This statement gives us the speed of Cars X and Y directly, so we can compute the difference in speeds through subtraction. T = 1 / 10 = 6 minutes
(2) SUFFICIENT. In three minutes, the distance between the two cars grew by 1/2 mile. R = 0.5 / 3 = 1mi / 6min. Thus, T = 6 minutes.

70
Q

List M (not shown) consists of 8 different integers, each of which is in the list shown below. What is the standard deviation of the numbers in list M?
4, 6, 8, 10, 12, 14, 16, 18, 20, 22
(1) The average (arithmetic mean) of the numbers in list M is equal to the average of the numbers in the list shown.
(2) List M does not contain 22.

A

OG13, HW 5, Q 109. Answer C. Statistics. In order to answer this, we need to know what is required to calculate the standard deviation of a list of numbers. The standard deviation is just a measure of how spread out the numbers on a list are relative to the mean. The formula for standard deviation is: SD = √∑(x - µ)^2 / N. The important takeaway from this formula is that we need to know the values of all the numbers on the list in order to compute the list’s standard deviation.

(1) INSUFFICIENT. Average = (Last + First)/2 = (22 + 4)/2 = 13. However, knowing that M’s average value of 13 is not enough. As long as we remove two integers whose average is 13 from the list (e.g. 4 + 22 or 12 + 14), the average value of the remaining numbers will also be 13, but the actual numbers of the list will be different, so the SD will be different.
(2) INSUFFICIENT. We have no way of knowing what the second number removed is
(C) SUFFICIENT. If 22 is not on list M and M’s average is 13, the removed number MUST be 4, because the only way to keep M’s average the same as the given list’s average is to remove two numbers with an average value of 13. Thus, the two statements together allow us to calculate the SD

Takeaway:
-Need to know the values of all the numbers on a list in order to compute a list’s standard deviation

71
Q

What percent of the drama club members enrolled at a certain school are female students?

(1) Of the female students enrolled at the school, 40 percent are members of the drama club
(2) Of the male students enrolled at the school, 25 percent are members of the drama club

A

OG13, HW 6, Q 21. Answer E. Overlapping Sets. There are several ways to determine the percent of the drama club members that are female (e.g. using ratios, percents, or fractions), so do not worry about finding specific values. However, we still need either a percent or ratio of female members of the drama club to total members of the drama club.

(1) INSUFFICIENT. Knowing the percent of female students who are drama club members does not tell us the percent of drama club members who are female. For example, say there are 100 female students and 40%, or 40, of them are in the drama club. If the drama club has 200 members, then the percent would be 40/200 = 20%. But if the club has 400 members, then the percent would be 40/400 = 10%
(2) INSUFFICIENT. Knowing the percent of male students who are drama club members does not tell us the percent of drama club members who are female
(C) INSUFFICIENT. To demonstrate the information that we have been given, set up a double-set matrix. Percent of drama club members that are female students = 0.4F / (0.25M + 0.4F), which is unsolvable. Without knowing how many male and female students there are in the school, we have no way of knowing what percent of the drama club is female

Takeaway:
-You only get to choose one smart number (e.g. you cannot choose numbers for M and F above)

72
Q

A certain high school with a total enrollment of 900 students held a science fair for three days last week. How many of the students enrolled in the high school attended the science fair on all three days?

(1) Of the students enrolled in the school, 30 percent attended the science fair on two or more days.
(2) Of the students enrolled in the school, 10 percent of those that attended the science fair on at least one day attended on all three days

A

OG13, HW 6, Q 34. Answer E. Overlapping Sets. From the question stem, we know that each student attended a science fair for either 0, 1, 2, or 3 days. Name variables such that D0 is the number of students who attended for no days, D1 is the number of students that attended for 1 day, D2 is the number of students that attended for 2 days, and D3 is the number of students that attended for 3 days. Rephrase the question to: “what is D3?”
D0 + D1 + D2 + D3 = 900
(1) INSUFFICIENT. 270 = D2 + D3. We cannot solve for D3 because we do not know D2
(2) INSUFFICIENT. We cannot solve for D3 because we do not know D1 or D2. We cannot solve by plugging this equation into the given equation either because we do not know the value of D0
0.1(D1 + D2 + D3) = D3
0.1D1 + 0.1D2 + 0.1D3 = D3
0.1D1 + 0.1D2 = 0.9D3
D1 + D2 = 9D3
(C) INSUFFICIENT. Even with all three equations, we cannot solve for D3 because we do not know the value of D0.

73
Q

How many people are directors of both Company K and Company R?

(1) There were 17 directors present at a joint meeting of the directors of Company K and Company R, and no directors were absent
(2) Company K has 12 directors and Company R has 8 directors

A

OG13, HW 6, Q 49. Answer C. Overlapping Sets. Set up a double-set matrix for each statement.

(1) INSUFFICIENT. Note that 0 people are neither directors of R nor K. This is because we know that 17 represents the total number of directors. Thus, each of the 17 people is a member of at least 1 board. No one is on neither board. Nonetheless, we cannot arrive at a value for the number of directors of both R and K.
(2) INSUFFICIENT. This is not enough information to solve for the missing value.
(C) SUFFICIENT. Now place all of the same information into the same matrix. From here, we can easily fill in every other box. 3 people are directors of both Company R and Company K. Notice that without zero in neither, we would not have sufficiency at this point.

R: K = ?, No K = 5, Total = 8
No R: K = 9, No K = 0, Total = 9
Total: K = 12, No K = 5, Total = 17

74
Q

At a certain company, a test was given to a group of men and women seeking promotions. If the average (arithmetic mean) score for the group was 80, was the average score for the women greater than 85?

(1) The average score for the men was less than 75
(2) The group consisted of more men than women

A

OG13, HW 6, Q 70. Answer C. Statistics. We know that a test was given to a group of men and women and that the average score for the group was 80. The question asks if the average score of the women was greater than 85. If we knew whether there were more women or more men, and we knew what the average of either the women or the men was, we would know whether the overall average of 80 was closer to the men’s average or to the women’s average.

(1) INSUFFICIENT. Look at two scenarios: (1) Suppose there is only 1 man and 1 woman in the group. If the man has a score of 74, then the woman must have a score of 86 for the average to be 80 – YES, Women Average > 85 (2) Suppose there is still 1 man but there are 2 women in the group. If the man has a score of 74, the women must have an average score of 83 for the total average to be 80 – NO, Women Average

75
Q

Is the number of members of Club X greater than the number of members of Club Y?

(1) Of the members of Club X, 20 percent are also members of Club Y
(2) Of the members of Club Y, 30 percent are also members of Club X

A

OG13, HW 6, Q 93. Answer C. Overlapping Sets. Set up a double-set matrix for each statement. Is X greater than Y?

(1) INSUFFICIENT. 0.2X are members of both club X and club Y. But we do not know whether X > Y
(2) INSUFFICIENT. 0.3Y are members of both club X and club Y. But we do not know whether X > Y
(C) SUFFICIENT. Both statements gave us information for the Yes-Yes cell. This is the key step: we can set those two expressions equal to each other and solve for X in terms of Y. Since X is 1.5 times the value of Y, X is in fact greater than Y. Even though we do not know the value of either X or Y, we know that X is greater than Y.
0.2X = 0.3Y
X = 1.5Y

76
Q

If successive tick marks shown on a number line are equally spaced, point x is three spaces to the right of 0, point y is four spaces to the right of x, and if x and y are the numbers designating the end points of intervals, what is the value of y?

(1) x = 1/2
(2) y – x = 2/3

A

OG13, HW 6, Q 112. Answer D. Consecutive Integers. By looking at the number line, we can write the following relationships. We can rephrase the question in any of several ways: “What is K?” or “what is X?” or “what is y – x?”
X = 3K
Y = 7K
Y – X = 4K

(1) SUFFICIENT. X = 1/2. If X = 1/2 = 3K, then K = 1/6 and Y = 7/6
(2) SUFFICIENT. If Y – X = 2/3 = 4K, then K = 1/6 and Y = 7/6

The tick marks represent an evenly spaced set, of which consecutive integer sets are a special example

77
Q

If N is a positive three-digit number that is greater than 200, and each digit of N is a factor of N itself, what is the value of N?

(1) The tens digit of N is 5.
(2) The units digit of N is 5.

A

CAT 2, Q 2. Answer A. Digits & Decimals. We are told that N is a positive three-digit number that is greater than 200 and that each digit of N is a factor of N itself. We must determine the value of N.

(1) SUFFICIENT. We are told that the tens digit of N is 5, thus N = x5z. All multiples of 5 end with a units digit of either 5 or 0. However, the units digit of N cannot be 0, since 0 is not a factor of any number. Therefore, the units digit of N must also be 5. Therefore N has the form x55, with only the hundreds digit left to consider. After working through the possible values of x, we can conclude that N = 555
- X = 2, N = 255. Not possible – X cannot be an even number because N is odd and thus does not have any even factors
- X = 3, N = 355. Not possible – N is not a multiple of 3
- X = 5, N = 555. This is a possible value for N
- X = 7, N = 755. Not possible – N is not a multiple of 7.
- X = 9, N = 955. Not possible – N is not a multiple of 9.

(2) INSUFFICIENT. We are told the units digit of N is 5, thus N = xy5. If the units digit of N is 5, then 5 must be a factor of N. All integers ending in 5 are multiples of 5, though, so this fact doesn’t narrow the possibilities any further.

N is a multiple of 5 and any number is a multiple of 1. Using only these digits, try to formulate two numbers that satisfy the statement. Both 515 and 555 satisfy statement (2) so it is not sufficient to answer the question.

78
Q

If n is a nonzero integer, is x^n < 1?

(1) x > 1
(2) n > 0

A

CAT 2, Q 7. Answer C. Exponents & Roots. The question asks whether x^n is less than 1. In order to answer this, we need to know not only whether x is less than 1, but also whether n is positive or negative since it is the combination of the two conditions that determines whether x^n is less than 1.

(1) INSUFFICIENT. If x = 2 and n = 2, x^n = 22 = 4. If x = 2 and n = -2, x^n = 2(-2) = 1/(22) = 1/4.
(2) INSUFFICIENT. If x = 2 and n = 2, x^n = 22 = 4. If x = 1/2 and n = 2, x^n = (1/2)2 = 1/4.

(C) SUFFICIENT. Taken together, the statements tell us that x is greater than 1 and n is positive. Therefore, for any value of x and for any value of n, x^n will be greater than 1 and we can answer definitively “no” to the question.

79
Q

Fourteen rectangular tiles form the outline of quadrilateral ABCD and inscribed quadrilateral EFGH. What percent of the area of rectangle ABCD is covered by the tiles?

(1) ABCD is a square.
(2) EFGH is a square.

A

CAT 2, Q 8. Answer D. Polygons. Let the length (longer dimension) of each rectangular tile be called L, and the width (shorter dimension) of each tile W. Then each horizontal side of square ABCD has total length 2W + 3L, and each vertical side has total length 4L.

(1) SUFFICIENT: Because ABCD is a square, these total lengths must be equal: 2W + 3L = 4L, which reduces to L = 2W. Therefore, each side of square ABCD is equal to 4L = 8W, and the total area of square ABCD is (8W)(8W) = 64W^2.

The total area of the tiles is 14(L × W) = 14(2W × W) = 28W^2. The desired fraction is thus (28W^2)/(64W^2) = 28/64.

(2) SUFFICIENT. Each horizontal side of square ABCD has total length 3L, and each vertical side has total length 4L – 2W. Because EFGH is a square, these total lengths must be equal: 3L = 4L – 2W, which reduces to L = 2W. Therefore, each side of square ABCD is equal to 3L = 6W.

In turn, ABCD must also be a square, since each of its sides is 2W longer than the corresponding side of EFGH (i.e., longer by W on each side). Therefore, each side of ABCD is equal to 6W + 2W = 8W, and the total area of square ABCD is (8W)(8W) = 64W^2.

The total area of the tiles is 14(L × W) = 14(2W × W) = 28W^2. The desired fraction is thus (28W^2)/(64W^2) = 28/64.

80
Q

Is the positive integer N a perfect square?

(1) The number of distinct factors of N is even.
(2) The sum of all distinct factors of N is even.

A

CAT 2, Q 10. Answer D. Divisibility & Primes.
(1) SUFFICIENT. The factors of any number N can be sorted into pairs that multiply to give N. However, if N is a perfect square, one of these ‘pairs’ will consist of just one number: the square root of N. Since all of the other factors can be paired off, it follows that if N is a perfect square, then N has an odd number of factors. This statement then implies that N is not a perfect square. (e.g. perfect square 25 has 3 distinct factors – 1 and 25, 5 and 5)

(2) SUFFICIENT. In order to test this statement, sum the number of distinct factors for several perfect squares.
4: factors are 1 and 4, 2 and 2. Sum of distinct factors = 7 (ODD)
16: factors are 1 and 16, 2 and 8, 4 and 4. Sum of distinct factors = 31 (ODD)
25: factors are 1 and 25, 5 and 5. Sum of distinct factors = 31 (ODD)
36: factors are 1 and 36, 2 and 18, 3 and 12, 4 and 9, 6 and 6. Sum of distinct factors = 91 (ODD)

Following this pattern, if N is a perfect square, then the sum of its distinct factors should be odd. However, we are told that the sum of the distinct factors of N is even. Thus, N is not a perfect square.

Takeaways:

  • If N is a perfect square, then it has an odd number of distinct factors
  • If N is a perfect square, then the sum of its distinct factors is odd
81
Q

A circle has two inscribed triangles. ABC is an equilateral triangle, and DAB is a right triangle. What is the area of the circumscribed circle?

(1) DA = 4
(2) Angle ABD = 30 degrees

A

CAT 2, Q 14. Answer A. Circles & Cylinders. We are told that DAB is a right triangle. A right triangle inscribed in a circle will have a diameter of the circle as its hypotenuse. Therefore DB is a diameter. Denoting the center of the circle with O, and adding the auxiliary line segment (radius) OA, we get a clearer picture.

We now see that both angle ADB and angle ACB are inscribed angles in the circle, and that they both span the same arc, ACB. Therefore, they must be equal in measure (and each equal to half the corresponding central angle, AOB. But we know how big angle ACB is—it’s 60 degrees, because ABC is an equilateral triangle. Thus, angle ADB is 60 degrees as well, and angle ABD must be 30 degrees. This is because the three internal angles of right triangle DAB add up to 180 degrees. At this point, we can recognize triangle DAB as a 30-60-90 triangle.

(1) SUFFICIENT. Knowing that the short side of the 30-60-90 triangle is equal to 4, we can solve for the hypotenuse DB, which is 8. We know that DB is the diameter of the circle, so we can calculate the desired area.
(2) INSUFFICIENT. This statement tells us nothing about the radius, diameter or circumference of the circle. In fact, in light of the advance work we did, we can see that it tells us nothing that we do not already know.

Takeaway:
-Two inscribed angles within a circle that span the same arc must be equal in measure (and each equal to half the corresponding central angle)

82
Q

If x and y are non-zero integers and |x| + |y| = 32, what is xy?

(1) -4x – 12y = 0
(2) |x| – |y| = 16

A

CAT 2, Q 17. Answer A. Linear Equations. Note that one need not determine the values of both x and y to solve this problem; the value of product xy will suffice.

(1) SUFFICIENT. The statement can be rephrased to say x = -3y. If x and y are non-zero integers, we can deduce that they must have opposite signs: one positive, and the other negative. Therefore, this last equation could be rephrased as |x| = 3|y|. We don’t know whether x or y is negative, but we do know that they have the opposite signs. Converting both variables to absolute value cancels the negative sign in the expression x = -3y. We are left with two equations and two unknowns, where the unknowns are |x| and |y|:
|x| + |y| = 32
|x| – 3|y| = 0

Subtracting the second equation from the first yields |y| = 8. Substituting 8 for |y| in the original equation, we can easily determine that |x| = 24. Because we know that one of either x or y is negative and the other positive, xy must be the negative product of |x| and |y|, or -8(24) = -192.

(2) INSUFFICIENT: Provides two equations with two unknowns. Solving these equations allows us to determine |x| = 24 and |y| = 8. However, this gives no information about the sign of x or y. The product xy could either be -192 or 192.
|x| + |y| = 32
|x| - |y| = 16

83
Q

The next number in a certain sequence is defined by multiplying the previous term by some positive constant k, where k ≠ 1. How many of the first nine terms in this sequence are greater than 1?

(1) The ninth term in this sequence is 81.
(2) The fifth term in this sequence is 1.

A

CAT 2, Q 24. Answer B. Formulas. The sequence described can be written as An+1 = kAn. Note that if the positive constant k is greater than 1, the sequence will be increasing, and if the positive constant k is less than 1, the sequence will be decreasing. Also note that k cannot equal 1, 0, or any negative number.

(1) INSUFFICIENT. The ninth term is 81. Consider the possible cases for k: k > 1 and k 1 and k 1, then the sequence will be increasing, meaning the terms one through four must be less than 1 and the terms six through nine must be greater than 1. In this case, again, a total of 4 terms will be greater than 1. Because 4 out of the 9 terms are greater than 1 regardless of the value of k, this statement is sufficient.

84
Q

If x, y, and z are integers greater than 1, and (3^27)(35^10)(z) = (5^8)(7^10)(9^14)(x^y), then what is the value of x?

(1) z is prime
(2) x is prime

A

CAT 2, Q 27. Answer D. Exponents & Roots. The best way to answer this question is to use the exponential rules to simplify the question stem, and then analyze each statement based on the simplified equation.
(3^27)(35^10)(z) = (5^8)(7^10)(9^14)(x^y)
(3^27)(5^10)(7^10)(z) = (5^8)(7^10)(3^28)(x^y)
(3^27)(5^10)(7^10)(z) = (5^8)(7^10)(3^28)(x^y)
25z = 3x^y

(1) SUFFICIENT. Analyzing the simplified equation above, we can conclude that z must have a factor of 3 to balance the 3 on the right side of the equation. Statement (1) says that z is prime, so z cannot have another factor besides the 3. Therefore z = 3.

Since z = 3, the left side of the equation is 75, so x^y = 25. The only integers greater than 1 that satisfy this equation are x = 5 and y = 2, so statement (1) is sufficient.

(2) SUFFICIENT. Analyzing the simplified equation above, we can conclude that x must have a factor of 5 to balance out the 5^2 on the left side. Since statement (2) says that x is prime, x cannot have any other factors, so x = 5. Therefore statement (2) is sufficient.

85
Q

If ab ≠ 0, is ab > a/b?

(1) |b| > 1
(2) ab + a/b > 0

A

CAT 2, Q 29. Answer C. Positives & Negatives.
(1) INSUFFICIENT. According to this statement, the magnitude of b is greater than 1; that is, either b > 1 or b < –1. On the other hand, a can have any value whatsoever (except zero). With that freedom, it’s possible to choose values of a and b that yield contrasting answers to the question.
If a = 1 and b = 2, then the answer is YES (2 > 1/2)
If a = –1 and b = 2, then the answer is NO (–2 < –1/2)

(2) INSUFFICIENT. In interpreting this statement, note the influence of signs. The values ab and a/b have to have the same sign (either positive or negative). In order for their sum to be positive, they must both be positive. Therefore, the two variables a and b are either both positive or both negative. Within these constraints, it’s possible to choose values of a and b that yield contrasting answers to the question.
If a = 1 and b = 2, then the answer is YES (2 > 1/2)
If a = 1 and b = 1/2, then the answer is NO (1/2 < 2)

(C) SUFFICIENT. Statement 1 indicates that b > 1 or b < –1. Statement 2 indicates that a and b are either both positive or both negative.

First, consider the case in which both a and b are positive. In that case, b > 1. Consider a the “starting” number. Multiplying a by a number greater than 1 will make a larger. Dividing a by a number greater than 1 will make a smaller. In other words, ab must be greater than a/b.

Now consider the remaining case, in which both a and b are negative. In this case, b < –1. Since both numbers are negative, the negative signs cancel when they are multiplied or divided—in other words, the outcome is exactly the same as in the case where both a and b are positive. Again, ab must be greater than a/b.

Therefore, ab > a/b in all cases. Both statements together are sufficient.

86
Q

If a, b, and c are positive numbers such that a is b percent of c, what is the value of c?

(1) a is c percent of b.
(2) b is c percent of a.

A

CAT 2, Q 30. Answer B. Fractions. We are told that a = (b/100) * c, which can be rephrased to say c = 100a/b. We must find the value of c
(1) INSUFFICIENT. a = (c/100) * b, which can be rephrased to say c = 100a/b. This statement is identical to the equation given in the question stem, so statement (1) does nothing to help determine the value of c.
(2) SUFFICIENT. b = (c/100) * a, which can be rephrased to say c = 100b/a. Set the equation from the question stem and the equation from the statement equal to each other. We learn that a = b, which means that c = 100
100a/b = 100b/a
100a^2 = 100b^2
a^2 = b^2 (we are told that a, b and c are positive)
a = b

87
Q

Refer to picture (Quant 2, Q 123). Triangle ABC is composed of two smaller triangles, ABD and BCD. Angles BCD and BDC are equal to 2x. Angle BAD is equal to x. In triangle ABC, what is the length of side BC?

(1) Line segment AD has length 6.
(2) x = 36

A

Quant 2, Lecture # 7, Q 123. Answer A. Triangles & Diagonals. The question is asking: BC = BD = ? We can use all of the information in the question stem to determine that angle ABD = x. Thus, BD = AD.

DBC = 180 – 4x
ABC = 180 – 3x
ABC = ABD + DBC
180 – 3x = ABD + 180 – 4x 
x = ABD

(1) SUFFICIENT. We are given the length of AD, which means that AD = BD = BC = 6
(2) INSUFFICIENT. Knowing the degree of x does not help us determine the length of side BC

88
Q

Refer to picture (OG12, Q 173). If arc PQR is a semicircle, what is the length of the diameter, PR?

(1) a = 4
(2) b = 1

A

OG12, Lecture # 7, Q 173. Answer D. Triangles & Diagonals. We are told that arc PQR is a semicircle, which provides two key pieces of information (1) PR is the diameter, which equals a + b (2) angle PQR is a right triangle. Thus, the two triangles shown in the picture are similar triangles because we are given two inscribed right triangles. We can relate the length of b to the length of a accordingly.

2/a = b/2
ab = 4

(1) SUFFICIENT. We are told that a = 4. Thus, b = 1 and PR = 5
(2) SUFFICIENT. We are told that b = 1. Thus, a = 4 and PR = 5

Takeaway:

  • There are two common disguises for similar triangles
    (1) Two inscribed right triangles (as shown above)
    (2) Acorn shaped triangles
89
Q

Can a certain rectangular sheet of glass be positioned on a rectangular tabletop so that it covers the entire tabletop and its edges are parallel to the edges of the tabletop?

(1) The tabletop is 36 inches wide by 60 inches long
(2) The area of one side of the sheet of glass is 2,400 square inches

A

OG13, HW 7, Q 42. Answer E. Polygons. We can rephrase the question to say “Are the length and width of the glass rectangle greater than or equal to the length and width, respectively, of the tabletop rectangle?”

(1) INSUFFICIENT. This statement tells us nothing about the size of the glass
(2) INSUFFICIENT. This statement tells us nothing about the size of the tabletop
(C) INSUFFICIENT. If the glass is 2,400 square inches, then it could be 40 inches wide and 60 inches long, since 40 * 60 = 2,400, so its dimension could be greater than or equal to those of the tabletop. However, the glass rectangle could also be 24 inches wide by 100 inches wide, since 24 * 100 = 2,400, which would mean that its width would be smaller than the tabletop’s width.

90
Q

In the xy-coordinate plane, is point R equidistant from points (-3, -3) and (1, -3)?

(1) The x-coordinate of point R is -1
(2) Point R lies on the line y = -3

A

OG13, HW 7, Q 74. Answer A. Coordinate Plane. Draw a picture to help visualize the solution.

(1) SUFFICIENT. If the x-coordinate of point R is -1, R is somewhere on the vertical line with the equation x = -1. Since the midpoint between the x-coordinates of the two points is -1, that line is the same distance from (-3, -3) and (1, -3), so any point on it is as well.
(2) INSUFFICIENT. If point R is on the horizontal line y = -3, then it could be the point (1, -3) itself, in which case it is closer to this point than to (-3, -3). Alternatively, point R could be the point (-1, -3), in which case it is equidistant to (-3, -3 and (1, -3)

91
Q

Refer to picture (OG13, Q 79). In the figure below, is the area of triangular region ABC equal to the area of triangular region DBA?

(1) (AC)^2 = 2(AD)^2
(2) Triangle ABC is isosceles

A

OG13, HW 7, Q 79. Answer C. Triangles & Diagonals. The question “Does Area(ABC) equal Area(DBA)?” can be rephrased to “Does ac = dx?”

(1) INSUFFICIENT. a = d√2 is not enough information because we do not know anything about the values of c and x
(2) INSUFFICIENT. a = c. Thus, we know that x = c√2, which is not enough because we do not know anything about the value of d.
(C) SUFFICIENT. From the first statement, we know that a = d√2, and from the second statement, we know that x = c√2. Start by replacing a with d√2

ac = (d√2)(c). Then we can rewrite the equation relating x and c in terms of c. 
x = c√2 
c = x/√2. Replace x with x/√2
ac = (d√2)(x/√2)
ac = dx
92
Q

Refer to picture (OG13, Q 155). In the rectangular coordinate system below, if OP

A

OG13, HW 7, Q 155. Answer A. Coordinate Plane. Redraw the picture exaggerating constraints (ie make sure that PQ is substantially longer than OP) so that our eyes will be working for us as we make sense of this diagram. We can rephrase the question as “Is (1/2)(b)(h) > 48 or is bh > 96?”

(1) SUFFICIENT. Knowing the coordinates of point P tells us the value of h = 8, the y-coordinate of P. It also tells us that OX = 6, the x-coordinate of P. First consider what would happen if OP = PQ. If OP = PQ, then triangle OPQ would be isosceles, and the height would be the perpendicular bisector of the base OQ. Thus, we have that OX = XQ = 6. However, we know that OP OX. Thus, the area of the smaller triangle is A = (1/2)(6)(8) = 24. In turn, we know that the Area of OPQ = (1/2)(something greater than 12)(8) = greater than 48.
(2) INSUFFICIENT. Knowing the coordinates of point Q tells us nothing about the height of triangle OPQ.

93
Q

Refer to picture (OG13, Q 166). What is the value of x + y in the figure below?

(1) w = 95
(2) z = 125

A
OG13, HW 7, Q 166. Answer C. Lines & Angles. The key is that the quadrilateral formed by the four lines has four interior angles that must sum to 360. Label interior angels p, q, r and s. With this information, we can rephrase the question to “what is the value of p + q?”
p + q + r + s = 360
x + p = 180
y + q = 180
(x + p) + (y + q) = 360 
x + y = 360 – p – q 

(1) INSUFFICIENT. w = 95, which means that 95 + s = 180 or s = 85. Unfortunately, this does not allow us to find the value of p or q, or the sum of p and q.
(2) INSUFFICIENT. z = 125, which means that 125 + r = 180 or r = 55. Unfortunately, this does not allow us to find the value of p or q, or the sum of p and q.
(C) SUFFICIENT. Since we know that s + r = 85 + 55 = 140, we can determine that p + q must equal 360 – 140 = 220. We have answered our rephrased question. x + y = 360 – 220 = 140

94
Q

If positive integer x is a multiple of 6 and positive integer y is a multiple of 14, is xy a multiple of 105?

(1) x is a multiple of 9
(2) y is a multiple of 25

A

OG12, Lecture # 8, Q 82. Answer B. Divisibility & Primes. Your first instinct in any divisibility problem should be to break down numbers into their prime factors. 105 = 3 * 5 * 7. Therefore, for xy to be divisible by 105, it needs to have at least one factor each of 3, 5 and 7 in its prime box. Because xy is the product of x and y, it must be divisible by the individual factors of x and y, namely 2 and 3 for x, and 2 and 7 for y. This assures us that the prime box of xy already contains at least one 3 and one 7 (as well as two 2’s, which are not relevant to the question at hand. The remaining question is whether xy also contains a 5. To answer this question with a Yes, we need to know that a 5 shows up in either x or y. Thus, we can rephrase the question as “is x or y divisible by 5?”

(1) INSUFFICIENT. The fact that x is divisible by 9 does not tell us whether it is also divisible by 5.
(2) SUFFICIENT. This statement tells us that y is divisible by 25. Any number that is divisible by 25 is also divisible by 5, since 5 is a factor of 25. Thus, y is divisible by 5, and we can answer the rephrased question

95
Q

What is the greatest possible common factor of a and b?

(1) a is an integer
(2) b = a + 4

A

MGMAT, Lecture # 8. Answer C. Divisibility & Primes. Note that we are looking for the greatest POSSIBLE common factor rather than the GCF itself. The greatest possible common factor between two integers is the distance between them.
(1) INSUFFICIENT. There are an infinite number of possibilities.
(2) INSUFFICIENT. We do not know whether or not a is an integer. The greatest common factor only applies to integers
(C) SUFFICIENT. If a is an integer, and b = a + 4, then b must also be an integer. The distance between a and b is 4 no matter what. The greatest POSSIBLE common factor between two integers is the distance between them. Thus, the greatest possible common factor is 4.

Takeaway:

  • Pay close attention to every word in divisibility problems; in this problem, the inclusion of the word “possible” completes changes the answer
  • If you were to exclude the word “possible”, then you would not be able to determine the GCF and the answer would be E
96
Q

A school administrator will assign each student in a group of n students to one of m classrooms. If 3

A

OG13, Lecture # 135. Answer B. Divisibility & Primes. We want to know if n students can be divided evenly into m classrooms. We know that m and n must both be integers because they are quantities, so this must be a Divisibility problem! This question can be rephrased as, “is n divisible by m?” given that 3 13

(1) INSUFFICIENT. This statement tells us that 3n is divisible by m (3n/m = integer). Choose smart numbers. If we pick a number for n first, it seems more straightforward to figure out what m could be (because m can only be between 3 and 13)

The question states that n > 13, so let’s try n = 14. The question states that 3n is divisible by m. If n is 14, then 3n = 3 * 14 = 42. If 3n is 42, what could m be? Of the numbers between 3 and 13, 42 is only divisible by 6 and 7. Now we know that, when n = 14, m can be 6 or 7. Now let’s go back to the original question. If n = 14 and m = 6, then n is not divisible by m (answer is NO). If n = 14 and m = 7, then n is divisible by m (answer is YES)

Why is this statement insufficient? When n = 14, 3 * 14 = 3 * 2 * 7. For (3 * 2 * 7) to be divisible by m, it must have all the prime factors that m has. That is why the only two possible values for m are 6 (2 * 3) and 7. Now we can see why 3n being divisible by m did not guarantee that n would be divisible by m. m is able to have a 3 as a prime factor that did not originally come from n.

(2) SUFFICIENT. Use the same methodology to see why statement 2 is sufficient. The statement tells us that 13n is divisible by m. The difference between the two statements is that there are possible values of m that are divisible by 3, but there are no possible values of m that are divisible by 13. Therefore, the only way that 13n can be divisible by m is if n contains all the prime factors of m. And that is just another way of saying that n must be divisible by m.

A quick look at a few numbers confirms that this statement is sufficient.

If n = 14, 13n = 13 * 2 * 7. Possible values of m include 7 (14 is divisible by 7)
If n = 15, 13n = 13 * 3 * 5. Possible values of m include 5 (15 is divisible by 5)

97
Q

There are four numbers that appear on a number line and are represented by letters q, r, s, and t (in that order). Of the four numbers represented on the line, is r closest to zero?

(1) q = -s
(2) –t

A

OG13, HW 8, Q 69. Answer A. Positives & Negatives. Sometimes, it is good to exaggerate the picture somewhat to ensure that we do not find unjustified features, such as even spacing

(1) SUFFICIENT. If q = -s, then q and s are equally spaced on opposite sides of zero. That is to say that zero is halfway between q and s (test numbers if unsure). If zero is halfway between q and s, r must be closer to zero than either one of them because it is definitely somewhere between q and s. If r is between 0 and s, then it is closer to 0 than s. But s and q are equidistant from 0, which means that r is also closer to 0 than q. The logic is the same if r is between q and 0. Finally, t is definitely farther from zero than s since t is to the right of s (and thus a larger positive number). Therefore, r is the closest to zero
(2) INSUFFICIENT. If –t

98
Q

If k is an integer such that 56 < k < 66, what is the value of k?

(1) If k were divided by 2, the remainder would be 1.
(2) If k + 1 were divided by 3, the remainder would be 0

A

OG13, HW 8, Q 83. Answer E. Divisibility & Primes. This remainders questions tells us that k is one of the numbers 57, 58, 59, 60, 61, 62, 63, 64, 65.

(1) INSUFFICIENT. This statement can be rephrased “k is not a multiple of 2” because all multiples of 2, otherwise known as even numbers, have a remainder of 0 when divided by 2. Since any integer that is not an even number is an odd number, the simplest possible rephrase to this statement is “k is an odd number”. Thus, k = 57, 59, 61, 63, or 65

(2) INSUFFICIENT. This statement can be rephrased as “k + 1 is a multiple of 3”. We can prove insufficiency by looking at the possible values of k and seeing that there are multiple possible values of k
59 + 1 = 60
62 + 1 = 63 are both multiples of 3

(C) In order to combine both statements, we can make a table of the odd numbers in the range and determine whether k + 1 is divisible by 4 for more than one of them

k, k + 1, is k + 1 divisible by 3
57, 58, N
59, 60, Y
61, 62, N
63, 64, N
65, 66, Y

If both statements are true, k could be either 59 or 65, so we do not have enough information to determine the value of k

99
Q

If x and y are integers, is x > y?

(1) x + y > 0
(2) y^x < 0

A

OG13, HW 8, Q 97. Answer C. Positives & Negatives.

(1) INSUFFICIENT. There is no information about the relative values of x and y. They could be equal, y could be greater than x, or x could be greater than y and this could still be true
(2) INSUFFICIENT. In order for y^x to be a negative number, y must be a negative integer and x must be an odd integer. This is not sufficient because x could be a negative odd integer, which does not have to be greater than y
x = -1, y = -1, y^x = -1, x = y
x = -1, y = -3, y^x = -1/3, x > y
(C) SUFFICIENT. Statement 2 tells us that y is a negative number. If x + y > 0, and y is negative, then x must be positive

100
Q

If x is negative, is x < -3?

(1) x^2 > 9
(2) x^3 < -9

A

OG13, HW 8, Q 101. Answer A. Inequalities. The important constraint here is that x is a negative number.

(1) SUFFICIENT. If x > 0, then x > 3. If x < 0, then x < -3. Since we are told that x is negative, we know that x < -3
(2) INSUFFICIENT. The tricky part of this statement is that the cube root of -9 is not an integer, so we cannot easily calculate the upper bound of x. However, we know that the upper bound of x must be between -2 and -3 because (-2)^3 = -8 and (-3)^3 = -27. This means that there are values greater than -3 that make the equation true. But it is also true that x can be less than -3. For instance, (-5)^3 = -125, which is less than -9. Therefore the statement is insufficient

101
Q

If x, y, and z are positive integers, is x – y odd?

(1) x = z^2
(2) y = (z – 1)^2

A

OG13, HW 8, Q 173. Answer C. Odd & Evens. For the Combo x – y to be odd, one of these terms must be odd and the other even. Thus, we can rephrase the question somewhat as “are x and y opposites in terms of Odds & Evens?”

(1) INSUFFICIENT. Create an odds/evens table.
z = odd, z^2 = odd, x = odd
z = even, z^2 = even, x = even

(2) INSUFFICIENT. Create an odds/evens table.
z = even, z – 1 = odd, x = odd
z = odd, z – 1 = even, x = even

(C) SUFFICIENT. Combine the tables. We can see that whenever x is even, y will be odd, and vice versa. Thus, x and y are opposite in terms of odds & evens, and x – y must be odd
z = even, z – 1 = odd, y = odd, x = even
z = odd, z – 1 = even, y = even, x = odd

Alternatively, we could use direct algebra by subtracting the y equation from the x equation:
x = z^2
y = z^2 – 2z + 1

x – y = z^2 – (z^2 – 2z + 1)
x – y = 2z – 1
2z must be even; thus, 2z – 1 must be odd, and x – y is odd as well

102
Q

Refer to picture (OG13, Q 122). In the figure, points A, B, C, D, and E lie on a line. A is on both circles, B is the center of the smaller circle, C is the center of the larger circle, D is on the smaller circle, and E is on the larger circle. What is the area of the region inside the larger circle and outside the smaller circle?

(1) AB = 3 and BC = 2
(2) CD = 1 and DE = 4

A

OG13, HW 9, Q 122. Answer D. Circles & Cylinders. Let R be the radius of the larger circle and r be the radius of the smaller circle. We are asked for the difference in areas of these circles, i.e. piR^2 – pir^2. Dividing by pi, we can rephrase the question as “what is R^2 – r^2”

(1) SUFFCIENT. AB is the radius of the smaller circle, so r = 3. Furthermore, AC is the radius of the larger circle, so R = 3 + 2 = 5. We thus have the values of both R and r directly
(2) SUFFICIENT. CE is a radius of the larger circle, so R = CD + DE = 1 + 4 = 5. Using the fact that all radii of a circle are equal, we can also establish that AC = 5. Since CD = 1, we know that the diameter of the small circle, AD, equals AC + CD = 5 + 1 = 6. Therefore, r = 3, and again we have both R and r.

103
Q

A box contains only red chips, white chips, and blue chips. If a chip is randomly selected from the box, what is the probability that the chip will be either white or blue?

(1) The probability that the chip will be blue is 1/5
(2) The probability that the chip will be red is 1/3

A

OG13, HW 9, Q 125. Answer B. Probability. Since there are only three chips, we can use the 1 – x principle to simplify the question. The only chips that are not white or blue are red, so we can rephrase as “what is the probability that the chip is red?”

(1) INSUFFICIENT. P(B) = 1/5 and P(W or R) = 4/5. However, we do not have any way of determining the probability that the chip is red. All we know is that this probability is between 0 and 4/5
(2) SUFFICIENT. This statement provides the answer to our rephrased question above. If the probability that the chip will be red is 1/3, the probability that the chip will be white or blue is 2/3.

104
Q

If 2 different representatives are to be selected at random from a group of 10 employees and if p is the probability that both representatives selected will be women, is p > 1/2?

(1) More than 1/2 of the 10 employees are women.
(2) The probability that both representatives selected will be men is less than 1/10

A

Quant2, HW 9, Q 122. Answer E. Probability. We know that m + w = 10. Thus, the probability that both representatives selected will be women is p = w(w – 1) / 90.
w(w – 1) / 90 > 1/2?
w(w – 1) > 45?
The question can be rephrased as “is the number of female employees 8, 9 or 10?”
(1) INSUFFICIENT. W > 5 is not enough to answer our rephrased question. W could be 6 or 7 (answers NO to the question) or w could be 8, 9, or 10 (answers YES to the question)
(2) INSUFFICIENT. m(m – 1) / 90

105
Q

If xy does not equal zero, what is the value of xy?

(1) (2/x) + (2/y) = 3
(2) x^3 – (2/y)^3 = 0

A

CAT #3, Q 3. Answer B. Linear Equations.

(1) INSUFFICIENT. Note that x and y do not have to be integers. Start by manipulating the equation by multiplying both sides by xy. The value of xy changes according to the values of x and y
xy[(2/x) + (2/y)] = 3xy
2y + 2x = 3xy
xy = (2y + 2x)/3

(2) SUFFICIENT. If we manipulate this equation and solve for xy, we come up with a distinct value for xy. 
x^3 – (2/y)^3 = 0
x^3 = (2/y)^3
x^3 * y^3 = 8
(xy)^3 = 8
xy = 2
106
Q

If list S contains nine distinct integers, at least one of which is negative, is the median of the integers in list S positive?

(1) The product of the nine integers in list S is equal to the median of list S.
(2) The sum of all nine integers in list S is equal to the median of list S.

A

CAT #3, Q 7. Answer A. Positives & Negatives. Because list S has nine integers (an odd number), the median must be the middle integer.

(1) SUFFICIENT: The median is an integer. According to this statement, the nine numbers must collectively multiply to this same integer. None of the numbers is a fraction, and all nine are different numbers. The only way to multiply a bunch of integers together and arrive at the same starting point is to multiply by 0 or 1. In order to use 1, every number on the list except one would have to equal 1; for example, 1 × 1 × 1 × 3 = 3. In order to use 0, though, only one number has to equal zero; for example, 1 × 18 × -3 × 0 = 0.

Therefore, the only way in which the product of nine different integers can equal just one of the integers on that list is if the product of the nine integers is zero. (Not convinced? Try to disprove that rule using some real numbers. If the list is -4, -3, -2, -1, 1, 2, 3, 4, 5 then the median is 1 but the product is definitely not 1. If the list is -10, -9, -8, -7, -6, -5, -4, -3, -2, the median is -6 but the product is not -6. If, on the other hand, the list is -4, -3, -2, -1, 0, 1, 2, 3, 4, then the median is 0 and the product is also 0.)

Further, since the product of the nine integers equals the median, the median itself must also be zero. The median, then, is definitely not positive; the statement is sufficient to answer the question.

(2) INSUFFICIENT: The median of list S is one of the nine integers in list S. Therefore, if the sum of all nine integers equals the median, it follows that the sum of the eight integers on either side of the median must be zero:
Sum of all nine integers = (Median) + (Sum of other eight integers)
Statement 2 says that the Sum of all nine integers equals the Median. Substitute that info into the above equation:
Median = (Median) + (Sum of other eight integers)
0 = Sum of other eight integers

This isn’t enough, though, to determine the median; in fact, any number can still be the median of the list, as long as the surrounding numbers sum to zero. For example, the list –10, –9, –8, –7, 5, 7, 8, 9, 10 satisfies the criterion and has a median value of 5; the list –10, –9, –8, –7, -1, 6, 8, 9, 11 also satisfies the criterion and has a median value of -1.

107
Q

List P contains m numbers; list Q contains n numbers. If the two lists are combined to produce list R, containing m + n numbers, is the median of list R greater than the median of list P ?

(1) The smallest number in list Q is greater than the largest number in list P.
(2) m = n

A

CAT #3, Q 8. Answer C. Statistics. Before looking at the statements, it is important to remember that nothing in the question assumes that the numbers in either list must all be different. For instance, if list P contains 3 numbers, {1, 2, 3} is one possible set, as is {1, 1, 1}.

(1) INSUFFICIENT. Under this condition, it is possible for the median of the combined list R to be greater than the median of set P. For instance, if list P contains the numbers 1, 2, and 3, and list Q contains the numbers 4, 5, and 6, then the median of list R (which contains all six numbers) is 3.5, which is greater than 2 (the median of list P).

It is also possible for the median of the combined set to equal the median of list P. For instance, if list P contains the numbers 1, 1, 1, and 1, and list Q contains the numbers 2, 2, and 2, then the median of list P is 1, and the median of list R is also 1.

(2) INSUFFICIENT. This information does not tell us whether the median of list R is greater than the median of list P. For instance, if list P contains the numbers 1, 2, and 3, and list Q contains the numbers 4, 5, and 6, then the median of list R (which contains all six numbers) is 3.5, which is greater than 2 (the median of list P).

It is also possible for the median of the combined set to equal the median of list P, as is perhaps most easily seen in cases in which all of the numbers are the same: e.g., if list P contains 1, 1, and 1, as does list Q, then the median of all three lists P, Q, and R is 1.

(C) SUFFICIENT. If both conditions are true, then the set will have two middle values: the largest value in list P and the smallest value in list Q. According to statement (1), these values are different, so the median, found by averaging them, will be distinct from both; the median will be greater than the value from list P, and less than the one from list Q. Since the median is greater than the largest value in list P, it must be greater than every value in list P, and so greater than the median of P; therefore, this statement is sufficient.

108
Q

If the original price of an item in a retail store is marked up by m percent and the resulting price is then discounted by d percent, where m and d are integers between 0 and 100, is the item’s final price (after both changes) greater than its original price?

(1) m > d
(2) m = 1.5d

A

CAT #3, Q 18. Answer E. Percents. Perhaps the most straightforward way to investigate this problem is by testing values. Since the problem deals with inequalities – the prompt question is an inequality, and there is also an inequality in one of the statements – we should pay particular attention to extreme values as test cases. In the following discussion, we will set the starting price to $100.

(1) INSUFFICIENT: Considering extreme cases, if m = 99 and d = 1, that’s a 99% markup followed by a 1% markdown. If the original price is $100, the price after the markup is $199, and the value of the discount is only $1.99. In this case, the final price is much greater than the original, so this is a YES answer to the question.

Can we find a NO answer? Considering the other extreme of d (large values), let m = 99 and d = 90. (We could investigate d up to 98 if necessary, but it’s best to stick to numbers that provide easy calculation.) With these values, the price after the markup is $199. The discount is 90% of $199, which is very close to 90% of 200 (= $180). Therefore, the price will be marked down by almost $180, substantially less than the original $100 price. This is a NO answer to the question.

(2) INSUFFICIENT: Again, consider extreme cases. The smallest possible values of m and d are m = 3 and d = 2. (We can’t use d = 1 because then m will not be an integer.) In this case, the price after the 3% markup is $103; the discount is 2% of $103, or $2.06. If $2.06 is taken away from $103, the result is greater than the original $100, so this is a YES answer to the question.

Can we find a NO answer? The largest possible values of m and d are m = 99 and d = 66. (To find these values, set m to the largest possible integer, 99. Then solve for d, given the equation m = 1.5d. If you do not receive an integer value for d, then try m = 98 and so on.) In this case, the price after the 99% markup is $199. The subsequent discount is 66% of $199, which is very close to 66% of 200 (= $132); this discount is large enough to bring the final price far below the original price of $100. This is a NO answer to the question.

(C) INSUFFICIENT. Review the work already done, so as not to duplicate effort needlessly. The case m = 99 and d = 66 satisfies both statements 1 and 2, so that’s a NO answer to the question.

Can we find a YES answer? Consider smaller values that satisfy both statements, say m = 45 and d = 30. In this case, the markup is 45%, taking the price from $100 to $145. The discount is 30% of $145, or $43.50. The final price is $145 - $43.50 = $101.50, which is greater than the original price; this is a YES answer to the question.

109
Q

In 2003 Acme Computer’s price for each of its computers was five times the price for each of its printers. What was the ratio of its gross revenue from computers to its gross revenue from printers in 2003?

(1) In the first half of 2003, Acme sold computers and printers in a ratio of 3:2; in the second half of 2003, Acme sold computers and printers in the ratio of 2:1.
(2) Acme’s 2003 price for each of its computers was $1,000.

A

CAT #3, Q 19. Answer E. Ratios. The question asks us to find the ratio of gross revenue from computers to that from printers, given that the price of a computer is five times the price of a printer.

(1) INSUFFICIENT: Statement (1) says that the ratio of computers to printers sold in the first half of 2003 was 3 to 2; one set of values satisfying this statement is 3 computers and 2 printers. Using example prices of $5 per computer and $1 per printer, we arrive at a gross revenue of $15 from computers and $2 from printers.

During the second half of 2003, the ratio of computers to printers sold was 2 to 1; one set of values satisfying this statement is 2 computers and 1 printer, grossing $10 and $1 respectively if we use the sample prices chosen above. With these numbers, then, the full-year gross revenues are $25 from computers and $3 from printers.

Alternatively, for the second half of 2003 Acme may have sold 4 computers and 2 printers, still in the required ratio of 2 to 1. In this case, Acme would have grossed $20 and $2 from computers and printers, respectively; with these numbers, then, the full-year gross revenues are $35 from computers and $4 from printers, yielding a different overall ratio. Therefore, statement (1) is insufficient to give us a definitive answer.

(2) INSUFFICIENT: Statement (2) tells us that one of Acme’s computers cost $1,000 in 2003, but it tells us nothing about the ratio or numbers of computers and printers sold.

(C) INSUFFICIENT: Statement (2) fixes the price of a computer at $1,000, but, if we inflate the sample prices from $5 and $1 to $1,000 and $200, both of the solutions given in the explanation of statement (1) will still be possible. (The arithmetic will be identical; the prices will be 200 times as great.) Therefore, statements (1) and (2) together are still insufficient.

110
Q

Any decimal that has only a finite number of nonzero digits is a terminating decimal. For example, 36, 0.72, and 3.005 are terminating decimals. If a, b, c, d and e are non-negative integers and p = (2^a)(3^b) and q = (2^c)(3^d)(5^e), is p/q a terminating decimal?

(1) a > c
(2) b > d

A

CAT #3, Q 26. Answer B. Digits & Decimals. For fraction p/q to be a terminating decimal, the numerator must be an integer and the denominator must be an integer that can be expressed in the form of (2^x)(5^y) where x and y are integers. (Any integer divided by a power of 2 or 5 will result in a terminating decimal.)

The numerator p, (2^a)(3^b), is definitely an integer since a and b are defined as integers in the question.

The denominator q, (2^c)(3^d)(5^e), could be rewritten in the form of (2^x)(5^y) if we could somehow eliminate the expression 3^d. This could happen if the power of 3 in the numerator (b) is greater than the power of 3 in the denominator (d), thereby canceling out the expression 3^d. Thus, we could rephrase this question as, is b > d?

(1) INSUFFICIENT. This does not answer the rephrased question “is b > d”? The denominator q is not in the form of (2^x)(5^y) so we cannot determine whether or not p/q will be a terminating decimal.
(2) SUFFICIENT. This answers the question “is b > d?”

111
Q

Each day at Dan’s new job, he earns $10 more than the day before. If he just completed his 15th day of work, how much did he earn on his first day?

(1) His total earnings for the three most recent work days equal $690.
(2) His total earnings for the three most recent work days exceed his total earnings for the first three work days by $360.

A

CAT #3, Q 30. Answer A. Consecutive Integers. Note that the daily increase is consistent: he always earns $10 more each day. This is essentially a consecutive integer problem (although, in this case, there are increments of 10, not 1). Use f to represent Dan’s earnings on the first day. Express his earnings on subsequent days as f + 10, f + 20, f + 30, … f + 140.

(1) SUFFICIENT: This statement provides the sum of the most recent three days:

(f + 120) + (f + 130) + (f + 140) = $690
3f + 390 = $690
3f = $300
f = $100

Dan earned $100 on his first day.

(2) INSUFFICIENT: This statement provides the difference between the totals for the first three days and the last three days:

First three days = (f) + (f + 10) + (f + 20) = 3f + 30
Last three days = (f + 120) + (f + 130) + (f + 140) = 3f + 390
Last three days − First three days = (3f + 390) − (3f + 30) = $360

This statement is always true, regardless of how much Dan made on the first day. (This information can be determined from the question stem alone!)

112
Q

If a, b, and c are integers and abc ≠ 0, is a^2 – b^2 a multiple of 4?

(1) a = (c – 1)^2
(2) b = c^2 – 1

A

CAT #3, Q 33. Answer C. Odds & Evens. Divisibility has to do with the products of numbers, so it is likely going to be better to use the (a – b)(a + b) form. For a product to be divisible by 4, either one of the values (a – b) or (a + b) must be divisible by 4 or each value (a – b) and (a + b) must be divisible by 2 (that is, even). For both (a – b) and (a + b) to be even, either both a and b must be even or both must be odd. We can use the properties of odds and evens to see this.

If we know that a and b have the same parity (that is, both are odd or both are even), then we know that the quantities (a – b) and (a + b) are even; the reverse is also true: if we know that either (a – b) or (a + b) is even, then we know that a and b have the same parity. Thus, we can prove sufficiency through any combination of these findings: (1) a and b have the same parity; (2) (a – b) is even OR (3) (a + b) is even.

(1) INSUFFICIENT: a = (c – 1)^2. Here we are given an algebraic expression for a, but no information about b. There is no way to prove whether (a – b) or (a + b) is even. Furthermore, the statement doesn’t tell us whether a is odd or even. To see this, we can pick numbers for c. If c were even (e.g. 2) then a would be odd (i.e. 1), or if c were odd (e.g. 3), then a would be even (i.e. 4). Thus, we can see that a will be odd if c is even and a will be even if c is odd.
(2) INSUFFICIENT: b = c^2 – 1. Here we are given an algebraic expression for b, but no information about a. There is no way to prove whether (a – b) or (a + b) is even. Furthermore, the statement doesn’t tell us whether b is odd or even. To see this, we can pick numbers for c. If c were even (e.g. 2) then b would be odd (i.e. 3), or if c were odd (e.g. 3), then b would be even (i.e. 8). Thus, we can see that b will be odd if c is even and b will be even if c is odd.

(C) SUFFICIENT: There are two ways to combine the information in the statements and prove sufficiency: recognizing a pattern involving odds and evens in the statements or manipulating the algebraic expressions.

In our individual examinations of statements 1 and 2, above, we determined that, if c is odd, then a must be even and b must be even. We also determined that, if c is even, then a must be odd and b must be odd. Either way, a and b must have the same sign, which is sufficient to answer the question. We can test either (a + b) or (a – b), but note that we don’t need to test both:

(a + b) = 2c2 – 2c = 2(c2 – c) = even, because anything multiplied by an even number (in this case, 2) is even. We can stop here because we only need to show whether (a + b) or (a – b) is even.

(a – b) = –2c + 2 = 2(–c + 1) = even, because anything multiplied by an even number (in this case, 2) is even. We can stop here because we only need to show whether (a + b) or (a – b) is even.

113
Q

Is x – 1/16 greater than 5/8?

(1) Four times the value of x is less than three.
(2) One third of two is less than the value of x.

A

CAT #3, Q 36. Answer E. FDPs. Rephrase this question by solving the inequality for x: “is x > 1/16 + 5/8?” or “is x > 11/16?”

(1) INSUFFICIENT: Translate this statement into math. 4x < 3 or x < 12/16

Some potential x values that are less than 12/16 are greater than 11/16 (x = 11.5/16 = 23/32, for example), but others are less than 11/16 (x = 0, for example). Therefore, it is uncertain whether x > 11/16.

(2) INSUFFICIENT: Translate this statement into math (1/3)(2) < x or x > 2/3

First, how do 2/3 and 11/16 compare? Which one is bigger?
Compare the two fractions by cross-multiplying, giving 3 * 11 = 33 for the left-hand fraction and 2 * 16 = 32 for the right-hand fraction. Because 33 is bigger than 32, 11/16 is bigger than 2/3.

Draw a number line and place both 11/16 and 2/3 on that number line. If x > 2/3, there are some values between 2/3 and 11/16 (that is, some values of x are less than 11/16). There are also some values greater than 11/16. Therefore, it is uncertain whether x > 11/16.

(C) INSUFFICIENT: Combining the information from the two statements gives a range for x:
2/3 < x < 12/16

Place these values on a number line: 2/3, 11/16, 12/16. Also draw in the range of possible values for x. This range still allows for values of x that are less than 11/16 and for values of x that are greater than 11/16. It is still uncertain whether x > 11/16.